N216 - Hematologic Disorders

Lakukan tugas rumah & ujian kamu dengan baik sekarang menggunakan Quizwiz!

Answer: B The patient has described the features of peripheral neuropathy and Lhermitte syndrome. These neurologic features in a patient with macrocytic anemia and hypersegmented neutrophils are indicative of vitamin B12 deficiency causing megaloblastic anemia, peripheral neuropathy, and subacute combined degeneration of the spinal cord

A 72-year-old woman presents with gradual onset of digital paresthesia and difficulty with fine movement of the hands. More recently, she has noticed an electrical sensation running down her trunk and into her limbs when she bends her head forward. She is found to have a hemoglobin concentration of 9.8 g/dL and a mean cell volume of 107 fL. A bloodfilm shows macrocytes, oval macrocytes, and hypersegmented neutrophils. Which of the following is the most likely diagnosis? A. Neuroacanthocytosis B. Vitamin B12 deficiency C. Folic acid deficiency D. Alcoholic liver disease E. Copper deficiency

Answer: C The patient has an acute presentation of eosinophilia following travel to a region likely endemic for parasites such as schistosomiasis. The next step would be a stool evaluation for parasites.

A middle-age women presents with progressive weight loss and diarrhea and is found to have blood eosinophil count of 5000 cells/mm3. She is a nonatopic individual and has recently returned from a trip to Brazil. Which of the following would be the next step in evaluating this person? A. Bone marrow analysis and cytogenetics B. HIV titer C. Stool culture and evaluation for parasites D. Colonoscopy E. Serologic testing for inflammatory bowel disease

Ankyrin

A peripheral membrane RBC protein that binds spectrin and links the actin cytoskeleton to the plasma membrane.

Answer: E The next logical step would be ruling out an infection. Serial complete blood counts should be performed but not in isolation because it has already been established that the patient's eosinophilia is persistent.

A well-appearing 60-year old women is found to have persistent blood eosinophil counts of 3500 cells/mm3. She denies risk factors for parasite infection, is nonatopic, and has started no new medication, although she has been taking a statin and antihypertensive agent for several years. Which of the following considerations should take place? A. Because this is a well-appearing female patient, there is little concern for hypereosinophilic syndrome. B. Complete work-up is indicated that includes a computed tomography scan of the abdomen and bone marrow analysis. C. Consideration of drug-induced eosinophilia is not appropriate because the patient has not started any new medications. D. The patient should be simply followed by serial complete blood counts because she is clinically well. E. A stool sample for ova and parasite should be the next step.

Activated protein C

Activity is enhanced by cofactor protein S functions as natural anticoagulant by deactivatating Factors Va & VIIIa (essential to coagulation cascade) Activated by thrombin-thrombomodulin complex

Answer: A Because she switched from a balanced diet to a vegetarian diet only 2 years ago, she is not at risk for dietary cobalamin deficiency; however, with prolonged vegetarianism for 5 years, she will need to be taking cobalamin supplements. Vegetarian diets are intrinsically low in bioavailable iron, but earlier she used to eat a balanced diet. Because carbohydrate (flour that makes bread and pasta) is the food source that is fortified with folic acid, her low-carb diet could be low in folate, even though she consumes "small servings of salads." Because she does not take folate supplements, the impending initiation of phenytoin, which interferes with folate absorption, places her at risk for having a baby with a neural tube defect. Recent studies show that taking folate before and during administration of phenytoin will reduce the possibility that she will develop gingival hyperplasia. Although folate deficiency does not cause neuropsychiatric problems in adults, iron deficiency is notorious for cognitive problems in young women with minimal (iron deficiency-induced) anemia.

An 18-year-old unemployed white woman is to be prescribed generic phenytoin for epilepsy. She is asymptomatic except for passing clots during her periods and "memory problems," and she has a hemoglobin level of 11 g/dL. She used to eat a balanced diet but has been on a vegetarian "low-carb diet" for the past 2 years ever since she learned two things from her favorite Hollywood actress: (1) her idol watches her weight by consuming small servings of salads and other organic-natural foods, and (2) she recently spoke out on a daytime television show about "our wanton exploitation of animals as a food source." The patient has lost 10 pounds in the past 8 months. She has had occasionally irregular periods and is concerned about pregnancy because her boyfriend does not like condoms. Which of the following statements is not true? A. This patient's anemia is likely to be due to a combined deficiency of iron, cobalamin, and folate. B. This patient is at risk for having a baby with neural tube defects because her diet is likely to be low in folate, and phenytoin can induce a folate deficiency. C. Taking folate before and during administration of phenytoin will reduce the possibility that she will develop gingival hyperplasia. D. The patient should be taking supplements of iron, cobalamin, and folate because her diet is likely to be low in these nutrients. E. At this time, the patient's memory problems can be attributed to iron deficiency but not to either folate deficiency or cobalamin deficiency.

Answer: C The clinical picture is most consistent with thalassemia intermedia. In the case of α-thalassemia, this would be most like hemoglobin H disease (most frequently resulting from the interaction of α+- and α0-thalassemia), with the β4 tetramers of hemoglobin H forming red cell inclusion bodies leading to chronic hemolysis. Patients with α- or β-thalassemia intermedia become severely iron overloaded from lifelong hyperabsorption of dietary iron, even without needing blood transfusions. A similar gut hyperabsorption of iron occurs in familial hemochromatosis states, but those individuals are not typically anemic. This patient's hyperglycemia, growth retardation, possibly her bone fragility, and even the history of a "weak heart" (perhaps due to cardiomyopathy) are all manifestations of systemic iron overload. Thalassemias are especially prevalent not only in people of Mediterranean and Middle Eastern origin but also in those from the Indian subcontinent. In contrast to thalassemia intermedia, a patient with β-thalassemia major would be expected to be transfusion dependent throughout life. Hydrops fetalis, in which all four α-globin genes are deleted, is incompatible with life. Hemoglobin E, even in its homozygous state, causes little if any anemia and microcytosis.

An 18-year-old woman from Pakistan has been observed for a lifelong severe, microcytic and hypochromic anemia. Her hemoglobin levels have generally been in the range of 7 to 10 g/dL, but she recalls needing blood transfusions only twice in her life, on both occasions after traumatic bone fractures. Throughout childhood, her parents had kept her from playing sports or participating in activities requiring significant physical exertion because they had been told by pediatricians that she had a "weak heart." She had markedly delayed puberty but now had regular menses. Physical examination shows marked hepatosplenomegaly; laboratory test abnormalities include indirect hyperbilirubinemia and hyperglycemia. What is the most likely diagnosis? A. β-Thalassemia major B. Homozygosity for hemoglobin E (HbEE) C. Hemoglobin H (HbH) disease D. Hydrops fetalis E. Hereditary juvenile hemochromatosis

syndromes of albinism and neutropenia due to defects in vesicular trafficking

Chédiak-Higashi syndrome Griscelli syndrome II Hermansky-Pudlak syndrome II

RBC membrane in disease state

In pathologic states, such as thalassemia, sickle cell disease, and diabetes, loss of phospholipid asymmetry with externalization of phosphatidylserine (amino phospholipid) leads to activation of blood clotting through conversion of prothrombin to thrombin and facilitates macrophage attachment to erythrocytes, marking them for destruction

clinical manifestations of primary myelofibrosis

Most patients present with anemia and marked splenomegaly. Contributing factors include ineffective hematopoiesis and hypersplenism. Spleen and liver enlargement in PMF is secondary to extramedullary hematopoiesis (EMH) and may be associated with hypercatabolic symptoms, including profound fatigue, weight loss, night sweats, and low-grade fever. Patients also experience peripheral edema; diarrhea; early satiety; and, occasionally, complications of portal hypertension, including variceal bleeding and ascites.

Diagnosis of sickle cell disease

Normocytic, microcytic, or macrocytic hemolytic anemia with reticulocytosis, increased levels of lactate dehydrogenase and aspartate aminotransferase, and a compatible clinical history

clinical manifestations of polycythemia vera

Persistent leukocytosis Persistent thrombocytosis Microcytosis secondary to iron deficiency Increased red blood cell mass JAK2 mutations Increased leukocyte alkaline phosphatase Splenomegaly Generalized pruritus (usually after bathing) Arterial and venous thrombosis, including unusual thrombosis (e.g., Budd-Chiari syndrome) Erythromelalgia (acral dysesthesia and erythema)

myeloproliferative neoplasms (MPNs)

Polycythemia vera (PV) essential thrombocythemia (ET) primary myelofibrosis (PMF) represent stem cell-derived clonal myeloproliferation with a propensity to evolve into acute myeloid leukemia (AML) also called blast-phase MPN

fanconi anemia

Rare genetic anemia from defects in DNA repair often have short stature with upper limb anomalies and hyperpigmented cafe au lait spots

Answer: A Chemotaxis is the attraction and oriented movement of neutrophils to an inflammatory site after the release of chemoattractants.

The attraction of neutrophils and other white blood cells to an inflammatory site is called which of the following? A. Chemotaxis B. Margination C. Diapedesis D. Phagocytosis E. Extracellular trap

MOST FREQUENT CAUSES OF LYMPHADENOPATHY IN ADULTS IN THE UNITED STATES

Unexplained Infection -In drainage area of infection (e.g., cervical adenopathy with pharyngitis) -Disseminated (e.g., mononucleosis, HIV infection) Immune disorders (e.g., rheumatoid arthritis) Neoplasms -Immune system malignancies (e.g., leukemias, lymphomas) -Metastatic carcinoma or sarcoma

Interferon-γ Receptor-1 Defects

When macrophages phagocytose mycobacteria, they produce IL-12, which in turn stimulates T cells to produce interferon-γ (IFN-γ). IFN-γ is critical to the killing of mycobacteria and other intracellular bacteria. autosomal recessive mutation = complete loss of receptor function autosomal dominant mutation = normal ligand binding but defective intracellular signal transduction

Agranulocytosis

a deficiency of granulocytes in the blood, causing increased vulnerability to infection. rare neutrophils and neutrophil precursors present, ratio of erythroid to myeloid cells is very high can be an idiosyncratic reaction to certain drugs and most likely involves immune suppression of granulopoietic progenitor cells.

secondary/acquired hypercoagulable states

a heterogeneous group of disorders that predispose to thrombosis by complex mechanisms.

lymphocytosis

absolute lymphocyte count of more than 5000/µL most common causes are viral infections such as Epstein-Barr virus and the hepatitis viruses

monocytosis

absolute monocyte count of greater than 500/µL usually occurs in the setting of chronic inflammation resulting from infections like tuberculosis, syphilis, or subacute bacterial endocarditis, autoimmune or granulomatous disease, and sarcoidosis can also be seen in malignancies, such as preleukemic states, nonlymphocytic leukemia including acute myelomonocytic and monocytic leukemia, histiocytosis, Hodgkin disease, non-Hodgkin lymphoma, and various carcinomas

Autoimmune Hemolytic Anemia (AIHA)

acquired autoimmune disease in which autoantibodies directed against autologous RBC membrane antigens lead to their accelerated destruction hemolysis mainly extravascular

Familial Mediterranean Fever

also known as recurrent polyserositis autosomal recessive autoinflammatory disease defect is a missense mutation in the MEFV gene, which encodes the protein pyrin. Pyrin is believed to be a transcription factor involved in downregulating inflammation, possibly through an effect on chemotaxis in neutrophils and monocytes results in a hyperinflammatory response characterized by abundant neutrophilic infiltration into the peritoneal, pleural, and joint spaces.

hemolytic anemia

anemia caused by a shortened lifespan of mature red blood cells (RBCs) in the peripheral circulation

Anemia of chronic disease or inflammation

anemia that occurs in the setting of a chronic disease state, usually one associated with elevated levels of inflammatory cytokines usually manifests as a normochromic normocytic process believed to be the second most common cause of anemia

classic triad of hereditary spherocytosis

anemia, jaundice, splenomegaly

pernicious anemia

autoimmune gastritis involving destruction of the gastric parietal cell mass leads to atrophy of the fundus and body of the stomach, absence of intrinsic factor and hydrochloric acid, and eventually severe cobalamin (B12) malabsorption and deficiency

Specific Granule Deficiency (SGD)

autosomal recessive disorder that manifests during infancy as the recurrent appearance of deep and superficial skin infections, respiratory infections, and abscesses. characterized by impaired neutrophil chemotaxis, reduced respiratory burst, and a defect in bacterial killing. Infections are commonly caused by S. aureus, Pseudomonas aeruginosa, and C. albicans.

Leukocyte Adhesion Deficiency (LAD-1)

autosomal recessive inherited disorder mutation in the gene encoding CD18 deficiency of β2 integrin receptors, which are required for neutrophil migration from the vasculature into the tissues Indications are high resting neutrophil counts accompanied by frequent dissemination, sepsis, and recurrent infections.

Diagnosis of AIHA

based on the presence of a positive result (reveals antibody-coated RNCs) on the direct antiglobulin test (DAT), also known as the direct Coombs test, and on the absence of any other hereditary of acquired cause of hemolysis standard antiglobulin reagent contains antibodies against all four classes of IgG and components of complement (usually C3 and C4)

Howell-Jolly bodies

basophilic inclusions seen in circulating RBCs in the absence of splenic function. The presence in peripheral blood indicates that the patient has undergone splenectomy or has a process that has rendered the spleen nonfunctional (e.g., sickle cell disease with repeated splenic infarcts, chronic graft-versus-host disease).

Diagnosis of anemia of chronic disease

both the serum iron and the transferrin saturation are reduced, reflecting dysregulation of iron homeostasis diagnosis can be made in the appropriate clinical setting after the exclusion of other causes

Folate deficiency

can arise from decreased supply (reduced intake, absorption, transport, or utilization) or increased requirements (from metabolic consumption, destruction, or excretion) nutritional deficiency most common cause alcoholism in US

Cobalamin malabsorption

can result from removal, bypass, or dysfunction of only 1 to 2 feet of terminal ileum b/c distal ileum has greatest density of cubam receptors also 2/2 metformin dosing

steps of neutrophil emigration to site of infection

capture and margination firm adhesion to the endothelial wall diapedesis

Sickle Cell Disease (SCD)

caused by a mutation in the β-globin gene (HBB) consists of a group of chronic hemolytic anemias characterized by vaso-occlusive events, hemolytic anemia, vasculopathy, widespread acute and chronic organ damage, and premature mortality

Hereditary Elliptocytosis

characterized by the presence of elliptical or oval cigar-shaped erythrocytes on peripheral blood smears of affected individuals most patients asymptomatic hallmark is normochromic, normocytic cigar-shaped elliptocytes on peripheral blood smear

RBC membrane proteins

classified as integral, penetrating or crossing the lipid bilayer and interacting with the hydrophobic lipid core, or peripheral, interacting with integral proteins or lipids at the membrane surface but not penetrating into the bilayer core.

Chronic Granulomatous Disease (CGD)

defect in NADPH oxidase → ↑ susceptibility to infections with catalase + organisms (S. aureus, Aspergillus, etc...)

neutropenia

deficiency of neutrophils may be due to decreased marrow production, increased margination (adherence to vascular endothelium and spleen) with or without sequestration by the spleen, or increased destruction of peripheral cells most commonly drug-induced

Cyclic neutropenia

defined as periods of neutropenia (≤200/µL) lasting 3 to 5 days and occurring at approximately 21-day intervals periods of neutropenia may be marked by recurrent fevers, mouth sores, and infections of the skin, upper respiratory tract, and ears disorder can be dominantly inherited or sporadic

Clinical manifestations in LAD-1

delayed separation of the umbilical cord, bacterial and fungal infections, delayed wound healing, impaired pus formation, and severe destructive periodontitis with rapid tooth loss. Patients usually die during childhood.

characteristics of aging erythrocyte

glucose utilization and ATP levels fall, leading to decreased membrane deformability and, ultimately, a shortened lifespan. Lower potassium levels, higher sodium levels, and decreased membrane lipids are also seen in ATP-deficient, aging erythrocytes.

Hepcidin

iron-regulated, acute phase reactant peptide that blocks both iron uptake in the gut and iron release from hepatocytes and macrophage stores

Antithrombin

major protease inhibitor of the coagulation system inactivates thrombin and other activated coagulation factors. produced in the liver

clinical manifestations of essential thrombocythemia

microvascular and vasomotor symptoms Major thrombosis major hemorrhage

hydroxyurea

sole drug approved by the U.S. Food and Drug Administration for treatment of sickle cell anemia increases HbF in most patients

Dyskeratosis congenita (DKC)

syndrome of nail dystrophy, leukoplakia, and skin pigmentation abnormalities with associated neutropenia or aplastic anemia, or both can be inherited in an autosomal dominant or recessive or X-linked fashion has been shown to be associated with mutations in several genes that are implicated in telomere maintenance with wide-ranging manifestations including pulmonary fibrosis and hepatic cirrhosis as well as bone marrow failure

clinical classification of β-thalassemias

thalassemia major (TM, transfusion dependent) thalassemia intermedia (TI, of intermediate severity, non-transfusion dependent) thalassemia minor (asymptomatic)

Hemophagocytic Lymphohistiocytosis and Macrophage Activation Syndrome

Genetic defects generally cause impairment of natural killer (NK) cell and cytotoxic T-cell function. Mutations tend to occur in the perforin gene or in genes important for the exocytosis of cytotoxic granules. diagnostic hallmark of HLH, although sometimes not found early in the course of the disease, is histopathologic evidence of hemophagocytosis in the bone marrow

spleen

the largest lymphatic organ in the body

Physiologic Antithrombotic Mechanisms

(1) antithrombin III (2) protein C and protein S (3) tissue factor pathway inhibitor (TFPI) (4) the fibrinolytic system.

Pure Red Cell Aplasia (PRCA)

-Anemia with decreased erythroid precursors in marrow -Granulocytes & megakaryocytes/platelets normal can be caused by parvovirus B19 infection, an agent that infects erythroid precursor cells and likely generates an erythroid suppressive immune response can also be mediated by T lymphocytes or natural killer cells that suppress cells of erythroid lineage can also develop as a complication of thymoma and is likewise caused by oligoclonal T-cell expansion that specifically suppresses erythroid progenitor cells. can also be associated with drug exposure (e.g., isoniazid, chlorpropamide, and phenytoin), lymphoid neoplasms (chronic lymphocytic leukemia) and myelodysplasia

Answer: A This child has a disease that is apparent in neither parent but has a sibling that died of the same disease. This is consistent with an autosomal recessive form of severe congenital neutropenia (SCN). SCN related to neutrophil elastase mutation is an autosomal dominant disorder. Cyclic neutropenia is a milder autosomal dominant disorder, also due to mutations in ELANE; it is clinical mild and rarely presents in infancy. Although it can appear sporadically, the occurrence in a sibling makes that virtually impossible. This case is most consistent with Kostmann syndrome, which is due to mutations in the HAX1 gene. Because it is a very rare recessive disorder, if the child lives to adulthood and has offspring, they will be obligate heterozygotes and, like the patient's parents, will have normal counts. Like patients with dominantly inherited SCN associated with neutrophil elastase mutations, these patients respond to G-CSF and have an increased risk for developing MDS/AML.

A 1-month-old baby boy presents with fever and cough and is found to have pneumonia associated with severe neutropenia. His parents have normal blood counts but had lost a baby girl at age 3 months under similar circumstances 2 years before. Which of the following statements is not true? A. The patient likely has a mutation in neutrophil elastase. B. The child should be treated with granulocyte colony-stimulating factor (G-CSF). C. The child is at risk for developing myelodysplastic syndrome and acute myeloid leukemia (MDS/AML). D. If the child survives, his offspring will probably have normal neutrophil counts. E. The child's white blood cell count likely has a 21-day cycle in and out of the normal range.

Answer: C The child has a congenital disorder of neutrophils that results in leukocytosis. The diagnosis is leukocyte adhesion deficiency. This disease arises from defects in leukocyte adhesion to endothelium. It can arise from defects in the integrin receptor common β chain (LAD-1), with loss of expression of leukocyte function-associated antigen 1 (LFA-1), C3bi receptor, and gp150;95, leading to a failure to ingest and kill microbes opsonized by C3bi. It can also arise from an abnormality of selectin glycosylation (LAD-2). ELANE and HAX1 mutations are causes of severe congenital neutropenia, and affected patients present with profound neutropenia. LYST mutations are the cause of Chédiak-Higashi syndrome, which is also associated with neutropenia. Congenital G-CSFR mutations are very rare, although two or three patients in the literature have been described to have such mutations as the basis for G-CSF-resistant severe congenital neutropenia. More commonly, G-CSFR mutations are somatic mutations that arise in patients with severe congenital neutropenia and may be a harbinger of the development of myelodysplastic syndrome or acute myeloid leukemia.

A 1-month-old infant presents with a temperature of 103° F and a white blood cell count (WBC) of 90,000. He has a history of delayed umbilical cord separation, poorly healing skin lesions, recurrent otitis media, and failure to thrive. He is admitted to the hospital, where he is documented to have methicillin-resistant Staphylococcus aureus (MRSA). He is placed on appropriate antibiotics, but he remains febrile, and his WBC is persistently elevated in the 80,000 to 100,000 range. This patient is likely to have a mutation in which of the following? A. ELANE B. HAX1 C. Integrin receptor β chain D. LYST E. G-CSFR

Answer: A Morphologically altered neutrophils are a strong indication of specific granule deficiency. This condition usually manifests during infancy, including recurrent skin infections and vitamin B12 deficiency. Treatment options include parenteral antibiotics and drainage of infections.

A 15-month-old female infant is admitted for recurrent deep skin infections. Flow cytometry confirmed deficiency in vitamin B12. Blood smear displays bilobed neutrophil nucleuses. What is the likeliest diagnosis? A. Specific granule deficiency B. Chronic granulomatous disease C. Interferon-γ receptor-1 defect D. Myeloperoxidase deficiency E. Macrophage activation syndrome

Answer: E The normal neutrophil count is partially determined by ethnic background. African American males frequently have a neutrophil count of 1000 to 1500, and African American females may have counts that are even lower. Many may actually have counts below 1000. With a total neutrophil count of 1120, a normal hematocrit and platelet count, and an unremarkable history and physical examination, the patient should be diagnosed with constitutional neutropenia. This requires no further evaluation, although if DARC testing is available, confirming she is Duffy negative will confirm the diagnosis. The patient should be reassured that these counts are normal for her and present no undue risk. She should leave for France as planned.

A 20-year-old African American college student receives a scholarship to spend her junior year abroad in France. She needs to have a health form completed before her departure. A complete blood count (CBC) reveals neutropenia, and her physician advises her to postpone the scholarship and to see you for evaluation. She is healthy with no history of excessive infections. Her physical examination is normal. CBC shows: white blood cell count 3400 w/30% neutrophils, hematocrit 41, platelets 200,000. What should you advise her? A. She should have a bone marrow to rule out a bone marrow process causing neutropenia. B. She should have serial blood counts to rule out cyclic neutropenia. C. She should have chromosome analysis of her peripheral blood. D. She should be evaluated for an elastase mutation. E. She should go to France without any further evaluation.

Answer: B None of these is an entirely satisfactory first-line therapy for aplastic anemia caused by an inherited bone marrow failure syndrome (Fanconi anemia, dyskeratosis congenita, Diamond-Blackfan anemia), as in this case. These patients do not respond to ATG and other immunosuppressive therapies (including cyclosporine). Growth factors like ESAs and G-CSF are generally ineffective in severe aplastic anemia. Hematopoietic stem cell transplantation (HSCT) that includes irradiation in the conditioning regimen exposes the patient to an additional risk factor interacting with the main underlying biologic defect of Fanconi anemia, the DNA repair process. The use of specifically bone marrow as the source of HSCT and reduced-intensity conditioning regimens with fludarabine appear to have the most favorable outcome at this point, although it will not reduce the subsequent risk of a second malignant neoplasm.

A 20-year-old man with severe aplastic anemia is diagnosed by chromosomal breakage testing to have Fanconi anemia. Which of the following would be the preferred first-line treatment? A. Antithymocyte globulin (ATG) B. Bone marrow transplant from a matched related donor after fludarabine-based conditioning C. Cord blood hematopoietic stem cell transplant after conditioning that includes irradiation D. Supportive therapy with erythropoiesis-stimulating agents (ESAs) and granulocyte colony-stimulating factor (G-CSF) E. Cyclosporine

Answer: B This syndrome is a severe form of preeclampsia/eclampsia, occurring during pregnancy or within 48 hours of delivery. The hemolytic anemia is microangiopathic in nature, and the detection of schistocytes is diagnostically useful.

A 24-year-old woman who was hypertensive during pregnancy has an epileptiform convulsion 24 hours postpartum. She has jaundice, edema, and proteinuria. An urgent blood count shows leukocytosis, neutrophilia, anemia, and thrombocytopenia. Her blood film confirms the thrombocytopenia and in addition shows left shift, toxic granulation, and red cell fragments (schistocytes). Which of the following is the most appropriate diagnosis? A. Disseminated intravascular coagulation B. Hemolysis, elevated liver enzymes, and low platelets (HELLP) syndrome C. Hemolytic-uremic syndrome D. Preeclampsia E. Thrombotic thrombocytopenic purpura

Answer D Acute anemia in patients with HbSS can have several causes, as described in the section Acute Anemia. A simple vaso-occlusive crisis should not be accompanied by an acute drop in hemoglobin and hematocrit. Sequestration crisis occurs in young children (peak age, 2 years) who still have a functioning spleen; they present with massive and painful enlargement of the spleen, not seen in this case, due to trapping of sickle cells in splenic sinusoids. Myelosuppression by hydroxyurea is always a concern, but her blood counts have been closely monitored and were unchanged just a week ago; and even if she happened to take an inadvertently high dose of the drug in the interval, her severe anemia should have been accompanied by major decreases in the white blood cell and platelet counts as well. Hemolytic crisis is essentially ruled out by the very low reticulocyte count. Aplastic crisis in HbSS patients is a rare but life-threatening emergency that is characterized by pure red cell aplasia typically as a result of parvovirus B19 infection. Parvovirus B19 has characteristic tropism for erythroid precursors and therefore causes severe erythroblastopenia in the bone marrow. Coupled with the ongoing brisk hemolysis of sickle cell disease, an abrupt shutdown of red cell production by the marrow leads to a precipitous fall in hemoglobin and hematocrit. In addition to aggressive but cautious red cell transfusions (to avoid fluid overload), intravenous immune globulin has been found to be effective by providing a large bolus of neutralizing antibodies to accelerate clearance of parvovirus B19.

A 25-year-old woman with sickle cell anemia (HbSS) is brought to the emergency department after an episode of syncope. She has been observed regularly in the comprehensive sickle cell center of the hospital and has been taking hydroxyurea 500 mg daily for the past 4 years, which has been associated with a marked decrease in painful sickle crises. Blood counts have been regularly monitored during the hydroxyurea therapy, and the most recent complete blood count 1 week ago showed the following: white blood cells, 7500; hemoglobin, 9.9; hematocrit, 28.5; mean corpuscular volume, 106; platelets, 320,000; reticulocytes, 6.5%. On examination in the emergency department, she is sluggishly responsive, is hypotensive, and has a thread pulse with a rate of 140. There is mild, diffuse abdominal tenderness but no organomegaly or mass palpable. Stat complete blood count now shows the following: white blood cells, 8500; hemoglobin, 3.8; hematocrit, 17.2; reticulocytes, 0.2%; platelets, 410,000. What is the most likely diagnosis? A. Acute vaso-occlusive crisis B. Sequestration crisis C. Marrow suppression by hydroxyurea D. Aplastic crisis E. Hyperhemolytic crisis

Answer: C Red cells normally contain 30-fold more folate than what is present in the serum. Therefore, hemolysis associated with malaria will release the 30-fold excess folate from red cells into the circulation and result in a falsely high serum folate value. Hence, the serum folate level is a poor test to use in this setting because it is falsely elevated in both malaria and cobalamin deficiency. In addition, the patient is likely to have a diet that is low in all three nutrients (folate, cobalamin, and iron); food frequency questionnaire-derived estimates from resource-limited countries (including Kenya) indicate that most women consume less than what is optimum. Nearvegetarian and vegetarian diets predispose to cobalamin and iron deficiency. The borderline nutritional cobalamin deficiency can also contribute to a falsely elevated folate level. Because the cornmeal she purchases has no additives (i.e., no folic acid), she will need to take folate supplements in preparation for pregnancy.

A 29-year-old Kenyan postgraduate year 2 medical resident at a major university hospital in the Midwest with known thalassemia minor is referred to you for advice. She was admitted a month ago with high fever and chills with drenching sweats. She had recently returned from a trip home, where she also got married. She mentioned that she had been bitten several times by mosquitoes during her honeymoon. Her dietary history is significant only for the fact that she cooks all her meals and subsists "mostly on ugali (organic cornmeal without additives) and kale or other sautéed vegetables, occasional (once per week) small servings of meat, and lots of tea." In the hospital, a diagnosis of falciparum malaria was made on the basis of finding parasites on a peripheral smear. The accompanying laboratory results showed a hemoglobin level of 6 g/dL, mean corpuscular volume (MCV) of 90, and corrected reticulocyte count of 15%; lactate dehydrogenase was high, and haptoglobin was low. The serum folate level was well within the normal range (12 ng/mL; normal, >2 ng/mL), the cobalamin level was borderline low (202 pg/mL), and the serum ferritin level was high (300 ng/mL). Although antimalarials led to an eradication of malaria, the patient has still not significantly improved her hemoglobin level above 9 g/dL 2 months later. All of the following are true except which one? A. The patient is at risk for deficiencies of folate, cobalamin, and iron because her native Kenyan diet is monotonous and relatively low in both animal-source foods and fresh produce. B. The patient could have had a Coombs-positive as well as a Coombs-negative hemolytic anemia associated with falciparum malaria and should be prescribed both antimalarials and micronutrients to support compensatory hematopoiesis. C. At presentation in the hospital, the folate value rules out the possibility of folate deficiency and argues against administration of folic acid. D. If the patient is taking long-term folic acid in planning for a pregnancy, the development of neurologic symptoms should immediately trigger concerns of a cobalamin deficiency. E. She should now be treated with iron, cobalamin, and folate to effect a complete resolution of the anemia and then be maintained with these supplements to optimize her nutrition.

Answer: A In view of the slowly progressive nature of the anemia, the characteristic findings of a megaloblastic anemia on the peripheral smear, and the known effect of metformin in interfering with trans-enterocytic transport of cobalamin (at the ileal level), there is no reason to proceed with a bone marrow examination. The serum cobalamin level is likely to be low after consumption of metformin for 5 years (this patient has been taking metformin for much longer—15 years); the serum folate level is likely to be high-normal among those consuming a balanced diet because of food fortification with folic acid (remember, he makes his own bread). The malabsorption of cobalamin in the stomach induced by a reduction of acid can, during several years, lead to symptoms of deficiency. Proton pump inhibitors and H2-blockers can induce malabsorption. However, the primary mechanism of metformin is believed to be at the level of cubam receptors in the ileum, where supplemental calcium (1.2 g/day) taken with the drug can prevent this well-recognized, potentially serious but reversible, delayed adverse effect of metformin.

A 58-year-old white man with type 2 diabetes for 15 years is evaluated for a 6- to 8-month history of slowly progressive fatigue, anemia, and mild peripheral neuropathy of both lower extremities. He had been well controlled with metformin 850 mg three times daily and only recently began taking a single dose of long-acting insulin at night because the latest hemoglobin A1c level was 10. He does not take supplements, and his diet has been well balanced. He regularly makes his own bread every 2 or 3 days. He has recently been prescribed proton pump inhibitors and an H2-blocker for dyspeptic symptoms. The hemoglobin level is 11 with an MCV of 108, with macro-ovalocytes and several five-lobed hypersegmented polymorphonuclear neutrophils seen on the peripheral blood smear. All of the following are true except which one? A. The patient has features of megaloblastic anemia and should have a bone marrow examination to confirm the diagnosis. B. The serum cobalamin level is likely to be low, but the serum folate level is likely to be in the high-normal range. C. This patient could have food cobalamin malabsorption because of inhibition of acid in the stomach. D. This patient could have cobalamin malabsorption at the level of cubam receptors in the ileum. E. Upper endoscopy is indicated to look for gastrinomas or gastric cancer if anti-intrinsic factor antibodies are found.

Answer: D Because this patient has celiac disease, in the short term, folate deficiency can develop; but in the long term, with more extensive intestinal involvement that includes the ileum, cobalamin deficiency can eventually develop. The existence of high methylmalonic acid and homocysteine values, although confirming cobalamin deficiency, does not rule out an associated folate deficiency, which is associated with high homocysteine values. Despite low ferritin values consistent with iron deficiency from celiac disease, the normal MCV probably reflects an underlying folate or cobalamin deficiency. Treatment with parenteral iron will unmask the hematologic features of megaloblastic anemia.

A 29-year-old white woman is referred because of easy fatigue and anorexia. During the past 5 years, she has noticed abdominal bloating and diarrhea that are closely linked to consumption of gluten-containing foods. There is pallor, and the hemoglobin level is 9 with an MCV of 90 and a serum ferritin level of 10 ng/mL. The serum anti-tissue transglutaminase antibody test result is positive. All of the following are true except which one? A. Treatment of the patient with total-dose parenteral iron will resolve the iron deficiency, but the anemia will likely persist. B. Treatment with iron alone will result in a progressive increase in MCV, which reflects unmasking of a megaloblastic anemia. C. The serum homocysteine and methylmalonic acid levels are likely to be increased and signify metabolic evidence of cobalamin deficiency. D. Folate deficiency is unlikely to be present in a patient with elevated methylmalonic acid and homocysteine from cobalamin deficiency. E. Folates are absorbed through the proton-coupled folate transporter in the jejunum, which can be involved in celiac disease.

Answer: A CHS is a generalized disorder of granule formation caused by abnormalities in the LYST gene, which is required for the correct trafficking of proteins into granules. Consequently, patients with CHS have other defects associated with abnormal granule formation, including oculocutaneous albinism, with decreased pigmentation and nystagmus, and abnormal-appearing neutrophil granules. As the disease is an autosomal recessive disorder, the heterozygous parents are usually totally unaffected, and siblings have a 25% chance of inheriting the disease. Patients with CHS often have a terminal phase of the disease associated with the development of hemophagocytic syndrome, manifesting as fever, splenomegaly, and pancytopenia, often as a result of Epstein-Barr virus infection.

A 3-year-old child presents with recurrent infections and a diagnosis of Chédiak-Higashi syndrome (CHS) is made. Which of the following is not true of his disease? A. He is likely to have associated nystagmus. B. His sibling has a 50% chance of having the disease. C. He is likely to have abnormal neutrophil granules. D. He is likely to develop hemophagocytic syndrome. E. He is likely to have a defect in skin pigmentation.

Answer: A The low neutrophil count is an indication of neutropenia, in addition to giant cytoplasmic granules in the blood smear, suggesting CHS. Significant indication of CHS includes partial albinism as seen in patients light pigmentation and defects in bacterial killing and chemotaxis, which can be confirmed through neutrophil function testing. Infections can be treated with prophylactic antibiotics.

A 3-year-old girl with abnormally light pigmentation of the hair and skin has a history of recurrent respiratory infection. Blood smear shows giant cytoplasmic granules and low neutrophil counts. What is the mostly likely diagnosis? A. Chédiak-Higashi syndrome (CHS) B. Specific granule deficiency C. Chronic granulomatous disease D. Interferon-γ receptor-1 defect E. Myeloperoxidase deficiency

Answer: E All of the disorders listed may be associated with a history of lifelong anemia. Pyruvate kinase and pyrimidine 5′-nucleotidase (P5N) deficiency are both recessively inherited (normal parents, affected sister.) Splenectomy failure is typical in P5N deficiency and some cases of hereditary stomatocytosis, in which splenectomy is contraindicated. Basophilic stippling of erythrocytes is a characteristic finding in P5N deficiency because of the accumulation of partially degraded, nondiffusable RNAs in the cell.

A 31-year old woman presents with a history of lifelong hemolytic anemia. Her parents are normal, but a sister also has anemia. She tells you she had a splenectomy as a teenager, but "it didn't work." Laboratory evaluation reveals hemoglobin of 10 g/dL, hematocrit of 31%, mean corpuscular volume of 96 fl, and reticulocyte count of 15%. The lactate dehydrogenase level is 750 IU/L. Peripheral blood smear shows evidence of hemolysis; many erythrocytes show prominent basophilic stippling. Her likely diagnosis is A. pyruvate kinase deficiency. B. glucose-6-phosphate dehydrogenase deficiency. C. hereditary spherocytosis. D. hereditary stomatocytosis. E. pyrimidine 5′-nucleotidase deficiency.

Answer: C The blood count is suggestive of iron deficiency, not β-thalassemia heterozygosity (in which moderately severe anemia would not be expected). The blood film shows the features of iron deficiency (hypochromia, microcytosis, and pencil cells) and of hyposplenism (acanthocytes, target cells, and Howell-Jolly bodies). The combination of hyposplenism and iron deficiency suggests a diagnosis of celiac disease with splenic atrophy

A 35-year-old pregnant woman complains of severe fatigue and swollen ankles. Her blood count shows the following: red blood cells, 4.22 × 1012/L; hemoglobin concentration, 7 g/dL; hematocrit, 0.29 L/L; mean corpuscular volume, 67 fL; mean corpuscular hemoglobin, 16.6 pg; mean corpuscular hemoglobin concentration, 24.5 g/dL. Her blood film shows hypochromia, microcytosis, and poikilocytosis including elliptocytes (pencil cells), target cells, and acanthocytes. Some Howell-Jolly bodies are present. Which of the following is the most likely diagnosis? A. β-Thalassemia heterozygosity B. Chronic blood loss C. Celiac disease D. Congenital sideroblastic anemia E. Dietary iron deficiency

Answer: B The patient is presenting with classic signs of hypereosinophilic syndrome and should be evaluated for FIP1L1-PDGFRA because its presence would lead to imatinib therapy. Delay of 6 months exposes the patient to the potentially unnecessary risk for eosinophilia-mediated organ damage.

A 40-year-old man presents with blood eosinophilia and rash. The chest radiograph is normal, and the blood eosinophil count is 5000 cells/mm3 on two occasions over the course of 1 month. He has no history of allergies and no risk factors for parasite infection. A bone marrow biopsy shows elevated eosinophils with no blasts. The next step would be which of the following? A. Lymphocyte phenotyping and serum immunoglobulin levels B. Determining the presence of the FIP1L1-PDGFRA fusion gene C. Following the eosinophil count over the course of 6 months to document true hypereosinophilia D. Determining IgE and HIV titers E. All of the above

Answer: A Cautious preoperative blood transfusion can improve surgical outcomes. Simple preoperative red cell transfusions to a hematocrit of about 30% are as effective as exchange transfusions to prevent postoperative complications, and they cause fewer transfusion-related complications. See the section Surgery and Anesthesia and reference A7 to the Transfusion Alternatives Preoperatively in Sickle Cell Disease ((TAPS) study, a randomized, controlled, multicenter clinical trial. Chronic iron chelation therapy is effective in the treatment of transfusional iron overload, but it is not urgent to begin it preoperatively, especially because he has not even had baseline magnetic resonance imaging evaluation for his level of iron overload

A 40-year-old man with sickle cell anemia (HbSS) requires open lung resection of a newly diagnosed and localized non-small cell lung cancer. He has had numerous hospitalizations in the past for sickle crises, including acute chest syndrome, and was taking hydroxyurea until about 3 years ago, when he discontinued it and was lost to follow-up. His complete blood count now is as follows: white blood cells, 11,000; hemoglobin, 9.5; hematocrit, 25.5; platelets, 420,000; reticulocytes, 4.5%. These values are similar to those he has had in the past. What preoperative management related to his sickle cell anemia should be recommended? A. Transfusion of red blood cells to a hematocrit of about 30% B. Exchange transfusions and initiation of iron chelation therapy C. Hydroxyurea with a loading dose and then his previous maintenance dose D. Erythropoietin therapy E. None of the above but close hemodynamic and hematologic monitoring

Answer: C Anti-intrinsic factor antibodies can interfere with the cobalamin-binding luminescence assays and lead to a failure in diagnosis of pernicious anemia in those with classic symptoms and sign (as in this patient). By contrast, antiparietal cell antibodies have no influence on this test. The identification of serum anti-intrinsic factor antibodies is highly specific and diagnostic for pernicious anemia; by contrast, antiparietal antibodies are only about 50% specific for pernicious anemia. The presence of cobalamin deficiency can falsely raise the serum folate level, which could drop below normal on replenishment of cobalamin. This patient probably has a cobalamin deficiency due to either vegetarianism (and even near-vegetarianism) or a combination of pernicious anemia with hypothyroidism and vitiligo (commonly associated autoimmune diseases); in addition to iron, replacement of cobalamin will be needed for a full hematologic response. A dimorphic picture due to iron deficiency and either folate or cobalamin deficiency is found in all the conditions listed in A.

A 42-year-old Asian woman presents with an 8-month history of progressive lethargy and fatigue, anhedonia, forgetfulness, menorrhagia, abdominal bloating and occasional diarrhea, paresthesias, and staggering in the dark. Physical examination reveals anemia with lateral tongue ulcers, mild vitiligo, thyromegaly, delayed ankle reflexes, and reduced vibration sense and proprioception. The hemoglobin level is 5 g/dL, the white blood cell count is 3000/mm3, and the platelet count is 130,000/mm3. The peripheral smear reveals a dimorphic picture with macro-ovalocytes and hypochromic microcytic red cells with marked anisocytosis and occasional teardrop cells. Several five-lobed and one six-lobed polymorphonuclear neutrophils are noted. Serum ferritin level is 5 ng/mL, serum cobalamin level is 800 pg/mL, and serum folate level is 5 ng/mL (lower limit of normal, <2.4 ng/mL). The thyroid-stimulating hormone concentration is 15 mIU/L. Which of the following is an incorrect statement? A. Vegetarianism, pernicious anemia with or without hypothyroidism, celiac disease, and pregnancy in women not taking supplemental iron or folate can give a similar picture of dimorphic red cells on a peripheral blood smear. B. Even without the Schilling test, it is possible to make the diagnosis of pernicious anemia with a noninvasive blood test. C. Antiparietal cell antibodies, which are diagnostic for pernicious anemia, can interfere with the current serum tests for cobalamin deficiency. D. The possibility of folate deficiency cannot be ruled out, given the current serum folate level. E. Treatment with a combination of iron and ascorbic acid plus L-thyroxine alone will not resolve this patient's anemia because it is likely she has an added dietary deficiency of cobalamin.

Answer: C Acute stress is commonly associated with leukocytosis. Acute leukocytosis in almost any setting occurs by demargination, in this case in response to the release of catecholamines during the stress of the accident, also causing his tachycardia. Cytokine-induced marrow proliferation and release of neutrophils reflect more chronic inflammation and could not have occurred in less than about a week. Unsuspected splenic rupture would likely lead to hypotension and shock. There is no reason to consider a diagnosis of chronic myeloproliferative disease or sepsis in the absence of other evidence for a secondary condition. The patient should have his counts rechecked in several days or weeks to rule out any other pathology.

A 45-year-old man presents to the emergency department after a motor vehicle crash. He is afebrile with a blood pressure of 160/95 mm Hg and a pulse of 110 beats per minute. He has several scrapes and bruises, but no major injuries. Admission complete blood count reveals: white blood cell count 16,000, hematocrit 45, and platelets 280,000. What is the most likely explanation for his leukocytosis? A. Acute bacterial infection B. Undiagnosed chronic myelogenous leukemia C. Stress-induced demargination of neutrophils D. Cytokine release stimulating increased marrow production and release of neutrophils E. Unsuspected splenic rupture

Answer: C This patient has severe warm antibody-mediated autoimmune hemolytic anemia (AIHA), which is probably related to her history of SLE. It appears to be sufficiently acute and severe to require transfusions of packed red blood cells, although the blood bank will likely have difficulty with typing and crossmatching. (The latter problem should not deter giving transfusions in a case like this in consultation with the transfusion medicine specialist on call, often having to resort to transfusing with not completely matched but the "best-matched" units of blood available.) High-dose corticosteroids are the mainstay of initial therapy. Rituximab, danazol, and splenectomy are all second-line treatment options. Plasmapheresis is usually ineffective because at least 50% of an individual's IgG (in this case, IgG autoantibody) is distributed into the extravascular space at any given time. Although azathioprine may have been effective in controlling this patient's other manifestations of SLE in the past, it is not good initial therapy for AIHA.

A 45-year-old woman who has been treated in the past for presently clinically inactive systemic lupus erythematosus (SLE) presents with a recent onset of fatigue, lightheadedness, and yellowness of her eyes. Her hemoglobin is found to be 6.3 g/dL (with a baseline of about 12.5-13.0), reticulocytes are 15%, indirect bilirubin is 2.8 IU/L, and LDH is 840 mg/dL. A peripheral blood smear shows a large number of polychromatic red cells and spherocytes. Her direct antiglobulin test (Coombs test) result is positive for IgG. Her treatment at this time should include A. plasmapheresis. B. rituximab plus danazol. C. intravenous methylprednisolone. D. urgent splenectomy. E. restarting her previous SLE treatment with azathioprine.

Answer: C Although erythropoietin injections have been documented to improve quality of life and performance status in patients with cancer and a hemoglobin of less than 10 g/dL who are undergoing chemotherapy, strong data supporting this therapy in patients with a hemoglobin of 10 g/dL or higher is lacking. The same is true for red cell transfusions, in the absence of symptomatic cardiovascular disease. In cancer patients not on chemotherapy, large meta-analyses of randomized trials have shown increased mortality and cancer progression in those treated with erythropoietin for anemia. In the absence of documented folate deficiency or sideroblastic anemia, there is no clear evidence that either folic acid replacement or pyridoxine therapy is of benefit in this setting.

A 55-year-old man with metastatic colon cancer presents 6 months after completing chemotherapy with complaints of fatigue. His complete blood count shows a hemoglobin level of 10 g/dL with microcytic, normochromic indices, and his serum ferritin level is in the high-normal range. He has no history or physical findings of cardiovascular disease. He requests some kind of treatment for his anemia. Which of the following is the most appropriate recommendation? A. Erythropoietin injections to increase his hemoglobin level B. Begin red cell transfusions to maintain a hemoglobin of 12g/dL or higher C. Advise that his fatigue may be somewhat affected by his hemoglobin level but that treatment to increase his hemoglobin level alone is unlikely to improve his symptoms and carries significant risks D. Oral folate replacement E. Oral pyridoxine therapy

Answer: A This patient's low levels of glutathione synthetase suggest a mild form of glutathione synthetase deficiency. Diagnosis can be confirmed with 5-oxoproline buildup and mutations in the GSS gene. Antioxidant supplementations are suggested for treatment.

A 6-year-old girl has a history of recurrent otitis and hemolytic anemia. Testing indicates significant excretion of 5-oxoproline in the urine and low amounts of glutathione synthetase in red blood cells. What is the mostly likely diagnosis? A. Glutathione synthetase deficiency B. Macrophage activation syndrome C. Chronic granulomatous disease D. Myeloperoxidase deficiency E. Hyper-IgE syndrome

Answer: B The clinical setting and laboratory findings in this patient are all consistent with coexistent anemia of chronic disease and iron deficiency. The normal ferritin level is most likely due to the chronic inflammation seen in patients with renal failure on hemodialysis. Increasing the dosage of erythropoietin in this setting in the absence of increasing available iron for erythropoiesis is unlikely to be beneficial and may introduce increased risk for thrombosis. Oral iron supplementation is usually ineffective in the setting of chronic inflammation due to elevated hepcidin levels that block iron uptake by the gastrointestinal mucosa. Intravenous iron therapy has been demonstrated to be effective in increasing the response to erythropoietin in patients with chronic renal failure. Although both iron dextran and iron gluconate are effective in this setting, iron gluconate has been shown to be safer.

A 60-year old man with chronic renal failure who is on hemodialysis has a falling hemoglobin level despite having been on erythropoietin (epo) therapy for a documented low serum epo level. His peripheral blood smear shows microcytic hypochromic red blood cells, and serum iron levels are low, but the serum ferritin level is 120 μg/dL. A bone marrow biopsy shows absent sideroblasts, and workup for occult blood loss is negative. Which of the following therapeutic interventions is likely to be the safest and most effective for increasing the patient's hemoglobin level? A. Oral iron supplement with ferrous sulfate B. Intravenous iron gluconate injections C. Doubling the dose of erythropoietin D. Intravenous iron dextran injections E. Oral iron supplementation with ferrous fumarate.

Answer: B The ferritin level is generally the best clinically available indicator of available iron for erythropoiesis. Although inflammation associated with arthritis could falsely elevate the estimation of iron stores associated with ferritin, this patient's arthritis is not highly active, as shown by the normal ESR and CRP level. Elevated CRP and ESR are indicative of even subclinical inflammation. Because these are normal, a ferritin level in this individual would likely indicate whether there is iron deficiency. Moreover, there are criteria for correcting ferritin levels in the presence of low or moderate inflammation.

A 60-year-old woman with a history of chronic arthritis that is currently under good control presents with symptoms of increased fatigue. She is noted to have a decreased hemoglobin of 8 g/dL and microcytic indices on her complete blood count. There is no evidence of blood loss by history or laboratory testing. The patient has a normal erythrocyte sedimentation rate (ESR) and a normal C-reactive protein (CRP) level. Which of the following routine laboratory tests is most likely to indicate whether this patient will respond to oral iron replacement with improvement in her hemoglobin level? A. Transferrin saturation level B. Serum ferritin level C. Serum iron level D. Vitamin B12 level E. Reticulocyte count

Answer: C Although it is not specific for polycythemia vera (PV), almost all patients with PV carry a JAK2 mutation, and its absence makes the diagnosis questionable. (The JAK2-V617F mutation also occurs in about 60% of patients with essential thrombocythemia and 50% to 60% with primary myelofibrosis.) Differentiating PV from other secondary causes of polycythemia (see Table 166-5) can be challenging if the individual is negative for a JAK2 mutation. Bone marrow aspirate and biopsy with cytogenetics can sometimes demonstrate characteristic findings of an myeloproliferative neoplasms, including trilineage hyperplasia (as opposed to just erythroid hyperplasia, the latter being what would be expected for the secondary polycythemias), as well as morphologic changes, but these are rarely absolutely diagnostic; furthermore, cytogenetic studies usually do not add to making or ruling out the diagnosis of PV. A red blood cell (RBC) mass determination is almost superfluous in this case because the great majority of patients with this degree of erythrocytosis, regardless of cause, will have elevated RBC masses. The finding of splenomegaly is neither specific nor sensitive for a diagnosis of PV, whether it is assessed by physical examination or by imaging. It would not be surprising to find an elevated level of carboxyhemoglobin in this smoker, but that would not be a conclusive finding for the diagnosis of his polycythemia. Bot mentioned among the choices, a serum erythropoietin would be a helpful ancillary test in a case like this; in general, it would be expected to be elevated in virtually all of the polycythemias but low in PV (the latter because of feedback inhibition of erythropoietin by the kidneys as a result of the primary myeloproliferative process of PV).

A 65-year-old man with coronary artery disease, hypertension, chronic obstructive pulmonary disease, and an active 40 pack year smoking history is referred by his primary care physician because of gradually increasing hemoglobin and hematocrit over the past 5 years. On physical examination, his spleen is not palpable. His blood counts now are hemoglobin, 18.9 g/dL; hematocrit, 60.5%; MCV = 78; white blood cell count, 11,200/µL (normal differential); and platelets, 485,000/µL. You are asked to evaluate the cause of this patient's polycythemia. Which of the following tests is most likely to either make or rule out the diagnosis of polycythemia vera in this case? A. Carboxyhemoglobin level B. Abdominal ultrasonography findings for spleen size C. PCR for JAK2-V617F D. Bone marrow aspirate and biopsy with cytogenetics E. Red blood cell mass

Answer: C The blood film features are those of hyposplenism, which also provides an explanation for the slight increase in the platelet count. The hyposplenism is the result of extensive amyloid deposition in the spleen. AL amyloidosis causes a variety of coagulation abnormalities, including acquired factor X deficiency, which could explain the prolonged prothrombin and activated partial thromboplastin times. Acquired hemophilia would not explain the hyposplenism.

A 67-year-old man presents with right heart failure and extensive bruising. The cardiac silhouette is normal on chest radiography. Electrocardiography shows reduced voltage of QRS complexes. Coagulation screen shows prolongation of the prothrombin time and activated partial thromboplastin time. A full blood count is normal apart from a slight increase of the platelet count to 487 × 109/L. A blood film shows Howell-Jolly bodies, target cells, occasional acanthocytes, and platelet anisocytosis. There is no relevant previous medical history. Which of the following is the most likely diagnosis? A. Essential thrombocythemia B. Hypothyroidism C. AL (light-chain associated) amyloidosis D. Acquired hemophilia E. Disseminated intravascular coagulation

Answer: E Although unusual, community-acquired M. pneumoniae infection can present with bilateral pulmonary infiltrates and acute respiratory failure. M. pneumoniae infection (as well as certain viral infections) are associated with cold-reactive autoantibody-mediated cold agglutinin and simultaneously hemolytic disease. This patient's abrupt onset of massive intravascular hemolysis could have been triggered by artificially cooling his body temperature. The patient's intravascular cold autoimmune hemolytic anemia (AIHA) is manifested by hemoglobinuria and his acute intravascular cold agglutinin disease by the development of acrocyanosis. The patient does not have gross hematuria; he has gross hemoglobinuria, so genitourinary tract disease cannot be the cause. Fulminant alveolar hemorrhage could cause a precipitous decline in hemoglobin but is very unlikely in the absence of bloody airway secretions via his endotracheal tube. Warm antibody-mediated AIHA secondary to cancer would not be expected to begin suddenly on hospitalization and would not explain the acrocyanosis. It would be too early for drug-induced hemolysis to begin in this way on the second day of admission.

A 68-year-old man is admitted to the intensive care unit because of acute respiratory failure secondary to clinically diagnosed community-acquired pneumonia. His temperature on admission is 41.2° C. Chest radiograph shows diffuse bilateral infiltrates. He is intubated and placed on a respirator, cultured, promptly started on broad-spectrum antibiotics, and placed on a cooling blanket. Intravenous hydration is begun. On the second day, the patient's condition deteriorates. Acrocyanosis develops in his fingertips and toes, the sclerae are noted to be icteric, and the urine from his Foley catheter has turned red (testing 4+ positive for heme but with only a small number of red blood cells). The admission hemoglobin of 14.2 has dropped to 7.8. What is the most likely cause of the acute anemia? A. Gross hematuria possibly secondary to ureteral trauma or a previously undiagnosed genitourinary malignancy B. Fulminant alveolar hemorrhage C. Warm antibody-mediated autoimmune hemolytic anemia most likely secondary to previously undiagnosed, widely metastatic lung cancer D. Drug-induced autoimmune hemolytic anemia, possibly caused by one of the antibiotics started on admission, with intravascular hemolysis E. Acute cold antibody-mediated autoimmune hemolytic anemia and agglutination secondary to Mycoplasma pneumoniae infection

Answer: A The first step is to promptly determine whether the apparently sudden drop in platelet count is real or artifactual. The abrupt drop (when counts were normal as recently as 1 day ago), accompanied by the striking absence of symptoms or signs of bleeding in this patient raises a high level of suspicion for "pseudothrombocytopenia." Seeing large clumps of platelets on the peripheral smear, which should be examined by the hematologist, would confirm the suspicion, which should be then promptly documented by ordering repeat platelet counts in both the customary purple-top tube containing EDTA as the anticoagulant along with tubes containing other anticoagulants (heparin or citrate). (The platelet count in the EDTA tube should be again very low, but not in blood collected into test tubes with the other anticoagulants.) Occasionally, pseudothrombocytopenia occurs as a result of the presence of a nonphysiologic "cold agglutinin," which clumps platelets in the test tube while sitting at room temperature before counting. This can be ruled out by having the laboratory do simultaneous platelet counts on a tube kept at 37° C versus one that has been intentionally left standing at room temperature: the former should have a normal platelet count if this is the case. These findings represent pseudothrombocytopenias in which the platelet counts are artifactually low and of no clinical consequence. These tests can be all done very quickly without stopping important medications, including heparin, and before undertaking further work-up and treatment of the thrombocytopenia if they remain necessary.

A 68-year-old man with coronary artery disease, diabetes mellitus, hypertension, and a history of stroke followed by seizures is now 3 days after an uncomplicated coronary artery bypass graft (CABG) surgery, and you are asked to consult for severe thrombocytopenia. The patient is on multiple medications for his chronic conditions and is now on postoperative subcutaneous heparin for thromboprophylaxis. There have been no bleeding problems postoperatively, and physical examination shows no signs of bleeding from external orifices and no ecchymoses or petechiae. The platelet count today is 5000, which triggered the STAT consult; it had been previously normal, including a count of 280,000 yesterday. Which of the following should be the first step? A. Examine the peripheral blood smear for platelet clumping. B. Stop heparin. C. Stop heparin and all other medications. D. Order STAT platelet transfusions. E. Take a thorough history and do a comprehensive physical examination.

Answer: C The most likely diagnosis is that this patient's PV is now transforming to either myelofibrosis or acute leukemia (or myelodysplasia). This is the natural history of the disease in a small minority of patients with PV even in the absence of previous treatments with known leukemogenic agents (e.g., P32, alkylating agents). A peripheral blood smear is important and can provide important clues, including leukoerythroblastic or myelophthisic changes (e.g., teardrop-shaped red blood cells, nucleated red cells, and immature myeloid precursors) could indicate development of fibrosis; the presence of circulating blasts could indicate conversion to acute leukemia. However, whether or not one sees these changes on the blood smear, a bone marrow aspirate and biopsy are indicated as the definitive test. There is no clear evidence that patients with myeloproliferative neoplasms (MPNs) are predisposed to second solid tumor malignancies; therefore, other than conducting a thorough history and physical examination and then pursuing any leads found with further testing, general body imaging to look for occult malignancy is not indicated. Clonal evolution does occur in PV and the other MPNs, and new mutations can be acquired during the course of the disease. However, at this point, it appears that the JAK2-V617F and CALR mutations are mutually exclusive in the MPNs. Even if stopping the hydroxyurea slightly improves the anemia, the striking history of increasing sensitivity to hydroxyurea suggests that something unfavorable is happening in this patient's bone marrow.

A 72-year-old woman has been followed by her hematologist for more than 15 years with the diagnosis of polycythemia vera (PV) (recently confirmed by the finding of the JAK2-V617F mutation). Her only treatments during this time have been periodic phlebotomies as needed to maintain a hematocrit below 45%, varying doses of hydroxyurea, and one aspirin daily. Her blood counts have been well controlled, and she has been largely asymptomatic. Over the past several months, despite lowering the hydroxyurea dose from 2 g/day to 1 g/day to 500 mg/day and now to 500 mg on alternate days, the patient has been noted to be actually anemic. Her only new symptoms are increasing fatigue. Her hematocrits have been ranging from 35% to 40% over this time. She reports no signs or symptoms of bleeding, her stools are negative for occult blood, and her serum ferritin level is within normal limits. What would you do now? A. Carefully examine the peripheral blood smear. B. Do evaluation for occult malignancy, including imaging of chest, abdomen, and pelvis. C. Perform bone marrow aspirate and biopsy. D. Check CALR (calreticulin gene) mutation status to rule out clonal evolution of disease. E. Stop hydroxyurea completely and observe course of the anemia.

Answer: D This patient has a moderate anemia that is not symptomatic, so it could have existed for many years undetected. Despite hypochromic and microcytic red cell features, there is no evidence of occult blood loss, and specific laboratory tests for diagnosing iron deficiency and inflammatory conditions were negative. The presence of ringed sideroblasts in an otherwise healthy young male with no history of drug or toxin exposure suggests that this may be a congenital X-linked sideroblastic anemia. The two major X-linked hereditary types of sideroblastic anemia result from mutations in either the erythroid-specific δ-aminolevulinic acid synthase gene, also known as ALAS-2, or the adenosine triphosphate-binding cassette gene (ABC-7). X-linked sideroblastic anemias are often mild with minimal or no symptoms and therefore may go undetected until a supervening process results in symptomatic anemia or they are detected incidentally, such as in this case. The presence of mild ataxia in this young male would be most consistent with a subgroup of patients with ABC-7 gene mutations who have associated ataxia.

A healthy 25-year-old man who was in a minor auto crash was found to have a microcytic, hypochromic anemia (hemoglobin 10 g/dL), unrelated to the accident, because there was no blood loss. The remainder of the complete blood count was normal. The patient was noted to be mildly ataxic but had no alcohol or drugs detectable on screening, and there was no head trauma. Serum iron level was normal, and serum ferritin level was in the high normal range. Workup for evidence of tumor or subclinical inflammation was negative, as was any history of prior drug or toxin exposure. A bone marrow showed normal hematopoiesis but demonstrated ringed sideroblasts. Which of the following tests or procedures is most likely to be useful for determining the underlying cause of this patient's anemia? A. Upper gastrointestinal endoscopy B. Genetic testing for mutations of the TMPRSS6 gene C. Serum transferrin receptor level assay D. Genetic testing for gene mutations of the genes coding for the adenosine triphosphate-binding cassette gene (ABC-7) and the ALAS-2 gene E. Serum folate level assay

Answer: E Glucose-6-phosphate dehydrogenase (G6PD) deficiency is inherited in an X-linked manner. Thus, if his wife does not have G6PD deficiency, all sons will be unaffected, and all daughters will be carriers (answer D). However, if his wife is a G6PD carrier, one of two daughters will be G6PD deficient, one of two daughters will be G6PD carriers, one son of two will be G6PD deficient, and one son of two will be unaffected. With the diagnosis of G6PD deficiency, he should always tell his health care providers, especially when they are prescribing medications, and he should avoid fava beans and all fava-derived products. Finally, he should seek medical attention when experiencing the symptoms of a hemolytic reaction, tiredness, shortness of breath, palpitations, and dark-colored urine.

A man recently diagnosed with glucose-6-phosphate dehydrogenase (G6PD) deficiency after an episode of hemolysis after taking chloroquine malaria prophylaxis before visiting Benin. He has many questions, including about starting a family. At the end of the visit, you provide several suggestions. These include all of the following except A. always inform health care providers that he is G6PD deficient, particularly before they prescribe any medication. B. seek medical attention if he is feeling tired, short of breath, and experiencing palpitations and if dark-colored urine is observed. C. he should avoid eating fava beans and fava plant-derived food. D. if his wife does not have G6PD deficiency, all sons will be unaffected, and all daughters will be carriers. E. if his wife is a G6PD carrier, one of two daughters will be G6PD deficient, one out of two daughters will be G6PD carriers, and his sons will not be affected.

Answer: C Unconjugated hyperbilirubinemia does not suggest acute liver failure. The blood film features are those of oxidant-induced hemolysis, and a diagnosis of glucose-6-phosphate dehydrogenase (G6PD) deficiency must therefore be considered. The blood film features are not consistent with the other types of hemolytic anemia listed. Most patients with acute hemolysis due to G6PD deficiency are men because this is an X-linked condition. However, hemolysis can occur in women because a proportion of red cells will be deficient. If there is marked lyonization, hemolysis may be severe. The G6PD assay is likely to be normal because the remaining red cells are not deficient and the G6PD concentration will be increased by the presence of reticulocytes. In this circumstance, the blood film is very important in suggesting the right diagnosis. A dietary history should be taken. It is possible that the patient has recently eaten broad beans

A young Greek woman presents to the emergency department because of the sudden onset of jaundice and fatigue. She has just returned from a brief trip to Greece for a family wedding. She is taking no medications. She is noted to be pale and has tachycardia. Her hemoglobin concentration is 8.7 g/dL and mean corpuscular volume is 109 fL. Reticulocyte count is increased to 280 × 109/L. Bilirubin is elevated and is found to be mainly unconjugated. Her blood film confirms marked anemia and shows polychromatic macrocytes, irregularly contracted cells, hemighosts (blister cells), and a few ghost cells. An assay for glucose-6-phosphate dehydrogenase is normal. Which of the following is the most likely diagnosis? A. Acute liver failure B. Autoimmune hemolytic anemia C. Glucose-6-phosphate dehydrogenase deficiency D. Hereditary spherocytosis E. Pyruvate kinase deficiency

Answer: A A reactive thrombocytosis is often seen after splenectomy, and it may persist for months or years. Toxic granulations in neutrophils are unrelated to removal of the spleen; they are seen in infections and inflammatory states regardless of spleen status. The other morphologic findings in blood cells are not modified according to whether or not there is an intact spleen.

After splenectomy, in addition to Howell-Jolly bodies, a peripheral smear is most likely to show which of the following? A. Thrombocytosis B. Toxic granulation of the neutrophils C. Spherocytes D. Dutcher bodies E. Nucleated red blood cells

Answer: B The most common organ involved in hypereosinophilic syndrome is the skin. Although this patient may have a drug reaction, the persistent cellulitis is concerning for an underlying primary inflammatory process.

An 18-year-old male presents with a generalized erythematous eruption and peripheral blood eosinophilia of 2500 cells/mm3. The history is significant for recent antibiotic use for a cellulitis in the leg. The cellulitis has not improved despite 10 days of antibiotics. The next step in evaluating this patient would include consideration of which of the following? A. The patient likely has a simple drug allergy and should switch antibiotics. B. The patient may have an underlying hypereosinophilic syndrome and should undergo skin biopsy, particularly at the site of the cellulitis. C. The patient should have a chest and abdominal computed tomography scan to rule out solid tumor malignancy. D. Referral should be made to a cardiologist to rule out cardiac involvement. E. It is critical to rule out parasitic infection as the next step.

Answer: D Eosinophilic esophagitis is a tissue-specific eosinophilic disorder that can only be diagnosed by endoscopic biopsy and does not have to be accompanied by peripheral blood eosinophilia.

An asthmatic young man presents with progressive difficulty swallowing that is refractory to proton pump inhibitor therapy for the past 2 months. The peripheral blood count and differential are normal, specifically without eosinophilia. Which of the following is true regarding eosinophilic esophagitis? A. It is not indicated because the patient does not have blood eosinophilia. B. It should only be considered after repeating the blood eosinophil count over a period of time. C. It can be assumed and treated with anti-IL-5 therapy. D. It should be ruled out with a referral to a gastroenterologist for endoscopic evaluation. E. It should only be considered after checking for the presence of the FIP1L1-PDGFRA fusion gene.

Answer: D This patient presents with classic routine laboratory findings of iron deficiency anemia. The failure to respond to oral iron replacement therapy suggests some reason for refractoriness to iron absorption. The patient is otherwise healthy, with no findings suggestive of malabsorption or inflammation or other reason to have high hepcidin levels that could account for this. However, the fact that intravenous iron supplementation only partially corrects the patient's anemia and iron deficiency suggests the presence of an excessive hepcidin level. Because the CRP and ACP levels were normal, demonstrating no evidence of inflammation to account for an elevated hepcidin level, it could be caused by a mutation in the TMPRSS6 gene, which encodes the matriptase-2 protein that acts to suppress excess hepcidin production. Mutations in TMPRSS6 can be either sporadic or familial.

An otherwise healthy 20-year-old woman with a history of excessive menstrual bleeding and increased fatigue was found to have a hemoglobin of 7 g/dL with microcytic, hypochromic indices, a low ferritin level, a low serum iron level, and reticulocyte count of 1.2%. Her menstrual bleeding pattern was normalized by hormone therapy, and she was begun on oral iron replacement with ferrous sulfate 350 mg three times daily. After 2 months, her hemoglobin was 8 g/dL, and there was no change in her other hematologic test results. After a full course of 2 g of intravenous iron gluconate, her hemoglobin, red cell indices, and iron level were partially corrected, but she remained somewhat anemic with a hemoglobin of 10 g/dL. A C-reactive protein (CRP) level and an ??1-acid glycoprotein (ACP) level were both normal. Which of the following laboratory tests would be most helpful to potentially explain her lack of complete response to iron therapy? A. Reticulocyte count B. Serum vitamin B12 level C. Bone marrow biopsy D. Genetic analysis for mutation in the TMPRSS6 gene E. RDW on automated complete blood count

major integral protein of the RBC

Band 3 has two primary functions, ion transport and maintenance of protein-protein interactions. Band 3 mediates chloride-bicarbonate exchange and provides a binding site for glycolytic enzymes, Hb, and the skeletal proteins ankyrin, protein 4.1 and 4.2

Answer: D FNA is a popular and usually accurate means of diagnosing an infection or carcinoma involving a lymph node, like metastatic tonsil carcinoma. However, unequivocal diagnosis and histopathologic subtyping of lymphomas (including Hodgkin and non-Hodgkin lymphoma, such as post-transplantation lymphoproliferative disorder) or granulomatous disease (such as the noncaseating granulomas of sarcoidosis) require cutting needle or excisional lymph node biopsy, which preserves histologic architecture.

Fine-needle aspiration (FNA) of a lymph node is a good way to diagnose which of the following? A. Castleman disease B. Hodgkin lymphoma C. Sarcoidosis D. Metastatic tonsil carcinoma E. Post-transplantation lymphoproliferative disorder

Answer: C Although the results of osmotic fragility testing may be similar in patients with hereditary stomatocytosis and hereditary spherocytosis, spherocytes are rarely seen on the peripheral blood smear in those with hereditary stomatocytosis. In addition to hereditary spherocytosis, spherocytes may be seen on the peripheral blood smear of patients with Heinz body hemolytic anemia, autoimmune hemolytic anemia, and liver disease.

Evaluation of a patient with hemolytic anemia reveals the presence of spherocytes on peripheral blood smear. Which disorder is NOT in the differential diagnosis? A. Heinz body hemolytic anemia B. Autoimmune hemolytic anemia C. Hereditary stomatocytosis D. Hereditary spherocytosis E. Liver disease

pathobiology of sickle cell disease

Globin, the protein portion of hemoglobin, harbors the iron-containing porphyrin heme ring and permits the molecule to operate efficiently in oxygen transport and its other physiologic functions. Mutations can alter the primary amino acid sequence of the globin polypeptide Sickle hemoglobin (HbS: α2β2S) is caused by an adenine (A) to thymidine (T) substitution (GAG → GTG) in codon 6 of the β-globin gene (HBB), resulting in replacement of the normal glutamic acid residue by a valine (Glu6Val) HbS polymerizes when it is deoxygenated, a property only of hemoglobin variants that have the HBB Glu6Val substitution. Critical amounts of HbS polymer within sickle erythrocytes cause cellular injury and lead to the phenotype of sickle cell disease, which is recognized by hemolytic anemia and vaso-occlusion.

Answer: E Hemoglobin E is one of the most common mutations in the world, especially in the Indian subcontinent and Southeast Asia. Its phenotype is that of a mild thalassemia, not an unstable hemoglobinopathy. It is of little if any clinical importance in its heterozygous form (HbE trait); even individuals who are homozygous for hemoglobin E (HbEE) usually have only mild anemia and microcytosis, without any clinical sequelae. Hemoglobin E can be mistaken for iron deficiency in the absence of further laboratory testing, but it is not known to be associated with iron deficiency.

Heterozygosity for hemoglobin E A. Is a rare form of unstable hemoglobinopathy B. Is associated with lifelong, low-grade hemolysis C. Causes iron overload from hyperabsorption of dietary iron D. Is frequently associated with iron deficiency E. Is of little if any clinical significance

Mechanism of Antibody-Mediated Red Blood Cell Destruction

IgG anti-RBC autoantibodies are opsonins; when bound to autoantigens on RBC membranes, they instigate phagocytosis of the cells by macrophages Using its Fcγ receptors, the macrophage can ingest an entire IgG-coated erythrocyte or transform it into a spherocyte (microspherocyte) by nibbling away its surface. Antibody-coated spherocytes are more vulnerable to osmotic forces than normal, unsensitized RBCs ultimately surrender to macrophages, especially in the splenic sinusoids, where blood flows sluggishly. The rate of hemolysis in AIHA depends on the amount of autoantibody on the RBC surface, the affinity and avidity of autoantibodies for the RBC autoantigen, and the number of macrophages in the environment of the antibody-coated erythrocyte

Answer: E The peripheral blood smear in aplastic anemia is generally normal, even in severe cases. This does not mean that the peripheral blood smear does not provide useful information about the etiology of pancytopenia in many cases. For example, hypersegmented polymorphonuclear leukocytes (A) suggest megaloblastic anemia (which in more advanced stages can cause pancytopenia, not just anemia). Nucleated (B) and teardrop-shaped (C) red cells suggest a "myelophthisic" process that involves invasion of the bone marrow by "foreign" elements (like metastatic cancer, fibrosis, or granulomas).

In a patient with pancytopenia, which of the following abnormalities in the peripheral blood smear would provide a useful indicator that the cause of the pancytopenia is aplastic anemia? A. Hypersegmented polymorphonuclear leukocytes B. Nucleated red cells C. Teardrop-shaped red cells D. Giant platelets E. None of the above

Answer: E Localized or disseminated lymphadenopathy can be caused by malignant lymphoproliferative diseases (including follicular lymphoma and Waldenström macroglobulinemia; benign proliferative diseases of the immune system (including Castleman disease); and infections by bacteria, mycobacteria (including tuberculosis, fungi, chlamydia, parasites, and viruses). In contrast, multiple myeloma is a plasma cell dyscrasia (as opposed to Waldenström macroglobulinemia, which is actually a malignant lymphoproliferative disorder associated with a paraprotein) and is not characterized by lymphadenopathy.

Lymph node enlargement would be unusual in a patient with which of the following? A. Follicular lymphoma B. Tuberculosis C. Castleman disease D. Waldenström macroglobulinemia E. Multiple myeloma

FEATURES OF SICKLE CELL ANEMIA

Painful episodes: associated with higher hemoglobin and beneficially affected by high HbF Acute chest syndrome: associated with higher hemoglobin and beneficially affected by high HbF Stroke: associated with lower hemoglobin and little affected by HbF Osteonecrosis: associated with higher hemoglobin and beneficially affected by high HbF Priapism: associated with lower hemoglobin and little affected by HbF Proliferative retinopathy: associated with higher hemoglobin and HbSC disease Splenic infarction and sequestration more common in HbSC disease Leg ulcers: associated with lower hemoglobin and beneficially affected by high HbF Gallstones Aplastic crisis due to B19 parvovirus Osteopenia: bone marrow hyperplasia Nutritional deficiencies: folic acid, zinc, calories Pneumococcal disease and sepsis Placental insufficiency

Answer: A A major cause of this PMF patient's massive splenomegaly is likely to be extensive "extramedullary hematopoiesis" (EMH) within the spleen. EMH occurs in myeloid metaplasia with or without myelofibrosis. It represents essentially "ectopic" sites of renew hematopoiesis, recapitulating the normal fetal state where hematopoiesis was not restricted to bone marrow but also occurred in other organs such as the spleen and liver. Therefore, histopathology of this patient's spleen is likely to demonstrate many blood cell precursors, including megakaryocytes. Fibrosis, hemophagocytosis, and infiltration with storage cells are not characteristic findings in PMF (or any of the other myeloproliferative neoplasms [MPNs]). Clonal lymphocytic infiltration can cause splenomegaly in the lymphomas but not in the MPNs, which are clonal myeloproliferative, not lymphoproliferative, disorders.

Massive, painful splenomegaly develops in a 70-year-old woman with an established diagnosis of primary myelofibrosis (PMF). She is JAK2-V617F mutation negative but CALR mutation positive. She has become transfusion dependent and has been unresponsive to medical management of the splenomegaly. After much consideration of risks and benefits, it is decided to do a splenectomy. Which of the following are you most likely to find in the spleen on pathologic examination in this case? A. Megakaryocytes B. Fibrosis C. Hemophagocytosis D. Clonal lymphocytic infiltration E. Storage cells

Answer: A Splenectomy early in life makes individuals particularly susceptible to sepsis with encapsulated organisms such as Streptococcus pneumoniae. However, there is little if any strong evidence for susceptibility to overwhelming systemic infections with other types of bacteria or viruses. This is the rationale for the recommendations for vaccination against S. pneumoniae and possibly also Haemophilus influenzae and Neisseria meningitidis before splenectomy.

Splenectomy early in life makes one particularly vulnerable to which of the following? A. Pneumococcal sepsis B. Bacterial endocarditis C. Salmonella sepsis D. Herpetic encephalitis E. Severe influenza

Answer: C By far the most effective way in which the anemic patient compensates for a reduction in red cell mass and oxygen carrying capacity is by elevation of red cell 2,3-DPG, which shifts the oxygen binding curve to the right and lowers oxygen affinity, thereby enhancing oxygen unloading to tissues. Erythropoietin production (answer E) is markedly enhanced in nearly all patients with moderate or severe anemia (except those with chronic renal failure), but the high levels of plasma erythropoietin are ineffective in boosting hemoglobin levels in those with impairment of red cell production. Increased resting heart rate and stroke volume (answer A) occur only in patients with severe anemia, and neither these cardiac changes nor decreased peripheral vascular resistance (answer D) are efficient modes of compensation.

What is the most effective way a patient compensates for anemia due to impaired red cell production? A. Increased heart rate and stroke volume B. Increased red cell oxygen affinity C. Decreased red cell oxygen affinity D. Decreased peripheral vascular resistance E. Increased production of erythropoietin

Answer: C This patient has recurrent bacterial infections and pneumonia, which suggest the possibility of an immunodeficiency. Significant levels of IgE present with normal complete blood count and platelets indicate hyper-IgE syndrome. Genetic testing of the STAT3 gene can be used to confirm diagnosis. Good prognosis seen in patients with administration of antibiotics, antifungal prophylaxis, and IgG infusions.

What is the probable diagnosis of an 8-year-old boy with severe dermatitis, recurrent bacteria skin infections, and pneumonia for evaluation of immunodeficiency? Initial tests reveal normal complete blood count and platelets, 40,000 IU of immunoglobulin E (IgE), and normal IgA, IgM, and IgG levels. A. Familial Mediterranean fever B. Chédiak-Higashi syndrome C. Hyper-IgE syndrome D. Chronic granulomatous disease E. Leukocyte adhesion deficiency

Answer: C The kidney is the primary site of erythropoietin production in humans and other mammals. The transcriptional regulation of the erythropoietin gene depends on the sensing of oxygen tension within a subset of renal interstitial cells at the boundary of the cortex and medulla. Note that sensing of arterial oxygen tension in the kidney (answer D) would not benefit the patient with anemia because arterial PO2 would likely be normal. The sensing of oxygen tension in the carotid body (answers A and B) regulates the rate of respiration. Regulation of the red cell mass by sensing blood viscosity would be maladaptive (answer E).

What physiologic mechanism determines the regulation of erythropoietin levels in the plasma? A. Sensing of arterial oxygen tension in the carotid body B. Sensing of intracellular oxygen tension in the carotid body C. Sensing of arterial oxygen tension in the kidney D. Sensing of intracellular oxygen tension in the kidney E. Sensing of blood viscosity in the kidney

Answer: A Hepcidin binds to and inactivates ferroportin, the transmembrane protein responsible for the export of iron from mucosal epithelial cells in the duodenum as well as macrophages in the bone marrow and liver. As a result, there is impairment of iron absorption from the gut and release of iron from macrophages.

Which of the following best characterizes the impact of hepcidin on iron homeostasis? A. Decreased release of iron from the duodenal enterocyte and decreased release of iron from the macrophage B. Increased release of iron from the duodenal enterocyte and decreased release of iron from the macrophage C. Decreased release of iron from the duodenal enterocyte and increased uptake of iron into the bone marrow D. Increased release of iron from the duodenal enterocyte and decreased uptake of iron into the bone marrow E. Increased release of iron from the duodenal enterocyte and increased uptake of iron into the bone marrow

Answer: A In most patients with chronic renal failure (answer A), there is damage to cells at the medullary-cortical boundary that produce erythropoietin. Therefore, serum erythropoietin levels in these patients are inappropriately low. The liver (answer B) produces much less erythropoietin than the kidney, and therefore liver disease does not significantly affect erythropoietin production. Renal cell carcinoma (answer D) sometimes causes elevated levels of serum erythropoietin but never decreased levels. Serum erythropoietin is markedly elevated in patients with aplastic anemia

Which of the following causes of anemia is best explained by inadequate production of erythropoietin? A. Uremia B. Chronic liver disease C. Iron deficiency D. Renal cell carcinoma E. Aplastic anemia

Answer: B Thrombosis is the major cause of morbidity and mortality in the myeloproliferative neoplasms, and it often presents in unusual intraabdominal sites such as hepatic vein thrombosis (Budd-Chiari syndrome) and portal and mesenteric vein thrombosis. Erythromelalgia is a striking and characteristic (but not specific) syndrome of microvascular vasomotor and occlusive disease, mostly affecting the dependent parts of the extremities, typically the feet and sometimes the hands. It is characterized by excruciating pain and tenderness in a patchy distribution, sometimes accompanied by corresponding patches of erythema and warmth. The large vessel peripheral pulses are usually intact. Erythromelalgia is generally very sensitive to prompt response to aspirin. Both marrow fibrosis and spontaneous transformation to acute leukemia are unusual but well-described feared complications of PV and ET. However, aplastic anemia is not associated with these diseases (in contrast to its association with a different bone marrow stem cell disease, paroxysmal nocturnal hemoglobinuria.)

Which of the following is not a known complication of polycythemia vera (PV) and essential thrombocythemia (ET)? A. Budd-Chiari syndrome B. Aplastic anemia C. Erythromelalgia D. Marrow fibrosis E. Acute leukemia

Answer: D When lymphadenopathy occurs in mononucleosis, HIV infection, or SLE, it tends to be generalized. The lymphadenopathy of cat-scratch disease can mimic that of a lymphoproliferative disorder but tends not to be as large and firm as in the case described here. Hodgkin lymphoma is the most likely diagnosis among these choices in an otherwise healthy young patient with a single prominent and firm enlarged node.

Which of the following is the most likely diagnosis in a patient with a 4-cm firm, nubby, nontender right lower cervical lymph node in an otherwise well 26-year-old female? A. Infectious mononucleosis B. Systemic lupus erythematosus (SLE) C. HIV infection D. Hodgkin lymphoma E. Cat-scratch disease

Answer: D In patients with ineffective erythropoiesis, there is erythroid hyperplasia in the bone marrow. Therefore, the bone marrow examination (answer E) is informative. Moreover, these patients have markedly enhanced destruction of these erythroid precursors and, as a direct consequence, elevations of serum (nonconjugated) bilirubin (answer B) and LDH (answer A). Unlike patients with hemolytic anemia, the reticulocyte count (answer C) is low in ineffective erythropoiesis. In contrast to these informative tests, serum erythropoietin (answer D) is not helpful because it will be elevated in nearly all patients with anemia irrespective of pathogenesis.

Which of the following laboratory tests would be least informative for establishing the presence of ineffective erythropoiesis in a patient with anemia? A. Serum lactate dehydrogenase (LDH) B. Serum bilirubin C. Reticulocyte count D. Serum erythropoietin level E. Bone marrow examination

Answer: A Chronic cold agglutinin disease occurs in the great majority of cases in adults older than the age of 50 years. It is generally mediated by a complement-fixing monoclonal IgM autoantibody directed at the red blood cell (RBC) I antigen. However, when a direct antiglobulin test (DAT) is performed, the result is typically positive only for complement that remains fixed to the RBC membrane, and the associated IgM is readily eluted at room temperature before it can be detected in vitro. The great majority of patients with chronic cold agglutinin disease are now recognized to have an underlying B-cell lymphoproliferative neoplasm, such as non-Hodgkin lymphoma or chronic lymphocytic leukemia. The liver, not the spleen, is the major site of extravascular hemolysis in this disease, so splenectomy is not effective treatment.

Which of the following statements about chronic cold agglutinin disease is incorrect? A. It occurs mainly in young women, sometimes with other autoimmune disorders. B. It is mediated by IgM autoantibody. C. The direct antiglobulin (Coombs) test result is positive for only complement on the red blood cell surface. D. It is associated with neoplastic B-cell lymphoproliferative disorders in most cases. E. Splenectomy not effective in its treatment.

Answer: A The highest risk indicators in hematopoietic stem cell transplantation for thalassemia are hepatomegaly, hepatic fibrosis, and poor iron chelation history. On the other hand, the Pesaro group's experience has demonstrated 87% disease-free survival in thalassemia patients who are younger than 17 years at the time of transplantation, have none of the risk factors noted, and are allografted from an HLA-identical related donor. Early experience with cord blood transplantation has shown it to be a safe procedure for thalassemia patients. Hematopoietic stem cell transplantation has been mainly successful in β-thalassemia. The risk of graft-versus-host disease (acute and chronic) after bone marrow transplantation from unrelated donors is at least as high in thalassemia as it is in other indications for bone marrow transplantation.

Which of the following statements is correct regarding hematopoietic stem cell transplantation for thalassemia? A. It can lead to thalassemia-free survival of at least 85% in low-risk, younger patients. B. It should not be administered with cord blood for transplantation. C. It has been mostly effective in the α-thalassemias. D. It is mostly reserved for thalassemia patients who are poorly iron chelated and have liver involvement. E. It is associated with a reduced risk of graft-versus-host disease compared with comparable unrelated transplants for other hematologic diseases.

Answer: E IVC thrombosis, a rare and dramatic form of venous thrombosis that can extend into the right atrium, is associated with renal cell carcinoma. Other characteristic (but not diagnostic) cancer-associated thrombosis syndromes include Budd-Chiari syndrome (hepatic vein thrombosis) or portal vein thrombosis in patients with myeloproliferative neoplasms, and superficial, migratory thrombophlebitis (Trousseau syndrome) with occult cancers of the gastrointestinal tract and pancreatic cancer. Subcutaneous low-molecular-weight heparin is superior to dose-adjusted warfarin for secondary prophylaxis after an episode of venous thromboembolism in a cancer patient, even when in remission, and it should be continued indefinitely. Although the incidence of occult malignancy in a previously healthy person who presents with unprovoked venous thromboembolism is definitely increased, most experts agree that a comprehensive history and physical examination, stool for occult blood, chest radiograph, blood counts and basic chemistries, and a mammogram in women is a sufficient initial screen for occult cancer, as long as any abnormalities picked up on that screen are more aggressively pursued. Increased risk for thrombosis in a patient with cancer continues for months or even years after complete remission has been achieved. An IVC filter is inadequate to prevent recurrent pulmonary embolism in patients with or without cancer.

Which of the following statements is correct regarding the relationship between cancer and thrombosis? A. In a patient being treated for gastric cancer, secondary prevention following a first episode of venous thromboembolism is best provided by warfarin for 3 to 6 months at doses to target an international normalized ratio of 2.0 to 3.0. B. A first episode of apparently unprovoked deep vein thrombosis in a previously healthy individual requires comprehensive search for an occult malignancy, including computed tomography of the chest, abdomen, and pelvis. C. Pulmonary embolism in a patient being actively treated for cancer requires the placement of an inferior vena cava (IVC) filter to prevent recurrence. D. A patient with breast cancer who completed treatment 6 months ago and is now in complete remission, with no evidence of disease, is not at increased risk for venous thromboembolism. E. In a previously healthy individual, the spontaneous development of extensive IVC thrombosis is highly suggestive of an occult renal cell carcinoma.

Answer: D Gene mutations for these three syndromes mostly involve inactivations of specific genes that encode the key proteins involved in their molecular pathogenesis, respectively, as correctly indicated in choices A to C. These also form the basis of screening tests for Fanconi anemia and dyskeratosis congenita; however, there are no adequate screening tests for Diamond-Blackfan anemia. (For Diamond-Blackfan anemia, serum adenosine deaminase is often elevated, but this does not represent a screening test.) Gene sequencing can provide specific mutation analysis for all three of these syndromes, when needed.

Which of the following statements is not correct regarding inherited bone marrow failure syndromes? A. Most of the known genes inactivated in Fanconi anemia encode proteins that protect the genome from excessive damage induced by chemical cross-linking agents. B. Most of the known genes inactivated in dyskeratosis congenita encode proteins that participate in the maintenance of telomeres. C. Most of the known genes inactivated in Diamond-Blackfan anemia encode ribosomal proteins. D. The best screening test for Diamond-Blackfan anemia is the chromosomal breakage test. E. The best screening test for dyskeratosis congenita is quantitative analysis of telomere length (flow-FISH) in lymphocytes.

Answer: C The type of bleeding complications this patient describes (i.e., delayed, deep bleeding, especially at oropharyngeal sites with surgery) is more characteristic of a coagulation factor defect than a platelet-vascular defect. His family history is suggestive of an X-linked inherited coagulopathy, with his mother being the silent carrier. Factor VIII deficiency (classic hemophilia) and factor IX deficiency fit that description. The PTT is not a sensitive screening test to pick up less severe forms of a coagulopathy (i.e., when the factor level is >25%), so mild hemophiliacs (with either factor VIII or IX deficiency) may have a normal PTT and yet still have serious bleeding complications when provoked with surgery. Therefore, factor VIII and IX levels must be obtained here to rule out the mild hemophilia, which would require prophylactic preoperative (as well intraoperative and postoperative) replacement therapy. The bleeding time, a method to test the integrity of platelet-vascular interactions, is now largely an obsolete test; and the kinds of bleeding problems this patient describes (as well as the notable absence of pathologic bruising with trauma) argues strongly against a qualitative platelet abnormality. Liver function tests could be done but are unlikely to shed additional light on this diagnostic problem; furthermore, this patient appears to have an inherited, not acquired coagulopathy. The possibility of von Willebrand disease is not unreasonable (even though it is usually autosomal dominant), although, again patients with von Willebrand disease tend to bleed superficially like patients with platelet problems. In any case, multimer analysis is not the first step to diagnosing von Willebrand disease: it should be preceded by a von Willebrand panel to assay levels of functional VWF, antigenic VWF, and factor VIII activity.

You are asked to clear a 48-year-old man for elective surgery for a herniated disc because he told his surgeon that he has a history of some kind of "blood problem." The patient tells you he bled heavily one day after a dental extraction 5 years ago, requiring blood transfusions in his local hospital's emergency department. He also recalls his parents telling him that he "almost died" of bleeding after a tonsillectomy in early childhood. There have been no other bleeding problems, even though he was an All-American soccer player in college and proudly says that he didn't bruise any more than his teammates. Family history is negative for bleeding except for his maternal grandfather, who had serious postoperative bleeding after coronary artery bypass graft surgery. The patient's screening coagulation test results now are normal: platelets = 380,000; prothrombin time (PT) = 11.4 (international normalized ration [INR] = 1.0); partial thromboplastin time (PTT) = 27.5. What would you do now? A. Clear him for surgery with close postoperative observation for bleeding. B. Order a bleeding time and platelet function tests. C. Order factor VIII and factor IX levels. D. Order liver function tests. E. Order von Willebrand factor (VWF) multimer analysis.

Answer: C Prominent stomatocytosis is common in patients with severe liver disease, and no additional specific therapy is indicated. Marked stomatocytosis is very common in advanced liver disease. Target cells may also be seen in advanced liver disease. Splenectomy is contraindicated in cases of hereditary stomatocytosis, not in cases of acquired stomatocytosis (answer A). Evaluation for hereditary stomatocytosis or hereditary spherocytosis via erythrocyte electrolytes (answer B), osmotic fragility or EMA binding (answer E) without additional historical data is not necessary. Although erythrocyte membrane lipids and cholesterol are perturbed in stomatocytosis, specific therapy is not warranted.

You are asked to see a jaundiced 62-year-old man with cirrhotic liver disease complicated by portal hypertension and hepatosplenomegaly because of stomatocytes on peripheral smear. He is being evaluated for splenectomy. His total leukocyte count is 4000/mm3 and his platelet count is 105,000/m3. Hemoglobin and hematocrit are 8.4 g/dL and 26%, respectively. Peripheral blood smear shows decreased numbers of leukocytes and platelets. Erythrocyte morphology reveals nearly 100% stomatocytes, erythrocytes with a central bar, and a few target cells. Your most appropriate response would be A. the presence of stomatocytes makes splenectomy an absolute contraindication. B. obtain erythrocyte electrolyte determinations. C. prominent stomatocytosis is common in patients with severe liver disease and no additional specific therapy is indicated. D. obtain cholesterol and triglyceride levels to correlate with severity of stomatocytes and target cells and consider lipid lowering therapy. E. obtain osmotic fragility or EMA binding test.

Answer: F Assessing spleen size and the biliary tract for the presence of cholelithiasis (answer A), determining if other family members could be affected after diagnosing a patient with hereditary spherocytosis (answer C), and counseling regarding hemolytic and aplastic crises (answer D) are all appropriate actions after diagnosing a patient with hereditary spherocytosis. Avoidance of oxidant-inducing food and medications (answer B), which is appropriate in patients with metabolic disease of the erythrocyte, especially glucose-6-phosphate dehydrogenase deficiency, is not necessary in hereditary spherocytosis.

You have just diagnosed a 23-year-old patient with hereditary spherocytosis. Which of the following are appropriate actions? A. Obtain ultrasonography of the spleen and gallbladder. B. Counsel the patient to avoid oxidant-inducing foods and medications. C. Determine if other family members could be affected and should be evaluated for the presence of hereditary spherocytosis. D. Counsel regarding possible occurrence of hemolytic and aplastic crises. E. A, B, and C F. A, C, and D G. All of the above

Hyperimmunoglobin E syndrome, or hyper-IgE syndrome

also referred to as Job syndrome group of genetically diverse, multisystem disorders of cytokine signaling characterized by recurrent skin abscesses, pneumonia, and periodontal diseases After birth, patients usually have moderate to severe dermatitis, eczematous skin eruptions, nonerythematous abscesses, pneumatoceles, and severe osteoporosis that can result in bone fractures organisms most commonly present at infected sites are Staphylococcus aureus, Haemophilus influenzae, Escherichia coli, and Candida albicans. Patients have elevated IgE levels (typically >1000 IU/mL) and eosinophilia.

Aplastic anemia

life-threatening syndrome characterized by failure of the bone marrow to produce peripheral blood cells and their progenitors. hallmark is bone marrow hypocellularity and hypoplasia of the erythroid, myeloid, and megakaryocyte lines bone marrow biopsy is required to establish the diagnosis. most cases are acquired

Glutathione Synthetase Deficiency

autosomal recessive disorder Glutathione, which is a potent antioxidant found in granulocytes, is required for a normal respiratory burst and bacterial killing. typically have recurrent otitis and hemolytic anemia. Mildly affected patients typically present with hemolytic anemia; in the moderate form, metabolic acidosis occurs; and patients with the severe form in addition develop central nervous system impairment, such as seizures, mental retardation, and ataxia.

pathogenesis of AIHA

complex multistep process involving not only the autoantibodies but also various effectors of the immune system, including the complement system, macrophages, and B and T lymphocytes IgG anti-RBC autoantibodies mediate the destruction of RBCs mainly by the process of extravascular hemolysis, mainly in spleen when lytic components of the complement system participate in the process (rare), the destruction of RBCs usually occurs directly within the circulation (intravascular hemolysis)

severe congenital neutropenia (SCN)

disorder of severe neutropenia with neutrophil counts of less than 500/μL, presenting in the neonatal period with recurrent bacterial infections. four inheritances: Autosomal dominant (ELANE mutation) Autosomal recessive (Kostmann syndrome; HAX2 mutation) X-linked (WASP mutation) Other rare defects (G-CSFR mutation, unknown)

DEHYDRATED HEREDITARY STOMATOCYTOSIS (XEROCYTOSIS)

exhibit contracted and spiculated RBCs, dessicytes, a variable number of stomatocytes, and target cells characteristic biochemical abnormality is a decreased potassium concentration and total monovalent cation content

megaloblastic anemia

group of disorders characterized by a distinct morphologic pattern in hematopoietic cells commonly due to deficiency of vitamin B12 (cobalamin) or folate deficiency in cobalamin or folate results in common biochemical feature of defect in DNA synthesis along with lesser alterations in RNA and protein synthesis, leading to state of unbalanced cell growth and impaired cell division. majority of megaloblastic cells have DNA values between 2 and 4 N because of delayed cell division. morphologically expressed as larger-than-normal "immature" nuclei with finely particulate chromatin, whereas relatively unimpaired RNA and protein synthesis results in large cells with greater "mature" cytoplasm and cell volume. microscopic appearance of nuclear/cytoplasmic asynchrony (or dissociation) is morphologically described as megaloblastic.

OVERHYDRATED HEREDITARY STOMATOCYTOSIS (HYDROCYTOSIS)

group of disorders is characterized by stomatocytes, erythrocytes with a mouth-shaped (stoma) area of central pallor on peripheral blood smear, severe hemolysis, macrocytosis (110-150 fL), elevated erythrocyte sodium concentration, reduced potassium concentration, and increased total Na+ and K+ content

clinical manifestations of folate deficiency anemia

hematologic (pancytopenia with megaloblastic bone marrow) cardiopulmonary (secondary to anemia) gastrointestinal (megaloblastosis with or without malabsorption) dermatologic (hyperpigmentation of the skin premature graying) infertility (sterility) psychiatric (primarily a flat affect) symptoms

Sideroblastic anemia

heterogeneous group of anemias distinguished by the characteristic finding of excessive mitochondrial iron in erythroblasts, as manifested by iron-laden, ringed sideroblasts in the bone marrow in the presence of moderate to severe anemia These disorders result from mitochondrial defects either in the biosynthesis of the heme porphyrin ring or in the metabolism of iron typically hereditary X-linked

Thalassemia syndromes

heterogeneous group of inherited hemolytic anemias characterized by deficient or absent production of one of the globin chains of hemoglobin - leads to hallmark imbalanced globin chain synthesis (six different types of globin chains (α,β,γ,δ,ε,ζ) are found in normal human hemoglobin at different stages of development) most common single-gene disorder worldwide most frequent in southeastern and southern Asia, in the Middle East, in the Mediterranean countries, and in northern and central Africa clinical manifestations (phenotype expressions) extremely variable/depend on degree of globin chain imbalance

β-Thalassemias

include a considerably heterogeneous group of disorders of hemoglobin synthesis, all of which are characterized by a reduced output of the β chains of adult hemoglobin depending on the β-globin gene defects and their interaction, the production of β-globin chains is quantitatively reduced to different degrees, whereas the synthesis of α-globin continues as normal, resulting in accumulation of excess unmatched α-globin chains in the erythroid precursors. The free α-globin chains are not able to form stable tetramers; they therefore precipitate in the erythroid precursors, forming inclusion bodies that damage the red cell membrane, thereby causing premature destruction of erythroid precursors in the bone marrow (ineffective erythropoiesis). Ineffective erythropoiesis leads to a sequence of events responsible for bone marrow expansion, anemia, hemolysis, splenomegaly, and increased iron absorption.

chronic inflammatory conditions associated with leukocytosis and neutrophilia

juvenile rheumatoid arthritis rheumatoid arthritis Still disease Crohn disease ulcerative colitis granulomatous infection chronic hepatitis

differential lymph node consistency

lymph nodes containing metastatic carcinoma are rock hard lymph nodes containing lymphoma are firm and rubbery lymph nodes enlarged in response to an infectious process are soft

principal defect in hereditary elliptocytosis

mechanical weakness or fragility of the erythrocyte membrane skeleton, mostly in spectrin

IL-1, TNF-α, and interferon-β

mediators of inflammation (pro-inflammatory cytokines) that act to increase erythroid progenitor apoptosis, downregulate erythropoietin receptors and antagonize pro-hematopoietic factors directly antagonizes action of erythropoetin

Interferon-γ (IFN-γ)

most potent inhibitory factor of erythropoiesis

structural abnormalities in erythrocytes

mutations that weaken the linkage between heme and globin, disrupt secondary (α-helical) structure, or introduce a charged or polar side group into the hydrophobic interior of the globin subunit.

glycophorins

next most abundant family of integral RBC membrane proteins provide most of the negative surface charge required by RBCs to avoid sticking to each other and to the vascular wall are involved in transmembrane signaling and carry receptors for Plasmodium falciparum, a number of viruses and bacteria, and several blood group antigens

Spectrin

peripheral membrane protein found in RBCs that cause their characteristic biconclave shape; connects proteins in the plasma membrane with actin in the cytoskeleton

granulocyte-macrophage colony-stimulating factor (GM-CSF) granulocyte colony-stimulating factor (G-CSF)

potent cytokine that influences the proliferation, survival, maturation, and functional activation of cells from the neutrophil-granulocyte lineage. In normal individuals, circulating levels are very low (<100 pg/mL) in conditions of stress, G-CSF levels can rise to 20 times baseline levels, thereby resulting in a rapid increase in circulating neutrophils

Severe Glucose-6-Phosphate Dehydrogenase (G6PD) Deficiency

prevalence associated with distribution of malaria and provides partial protection against malarial infection. White individuals subject to recurrent infections, whereas Asians or blacks with similarly reduced levels are not. G6PD is crucial for regulating the availability of NADPH for the respiratory burst. Gene mutations associated with G6PD are located on the distal arm of the X chromosome and are frequently identified as missense mutations results in recurrent bacterial infections, hemolytic anemia, and jaundice.

Heriditary spherocytosis

primary defect is the loss of erythrocyte membrane surface area caused by defects in erythrocyte membrane proteins, including α spectrin, β spectrin, ankyrin, band 3, and protein 4.2 inheritance in 2/3 patients is autosomal dominant Splenic destruction of poorly deformable spherocytes is the primary cause of hemolysis

margination

process by which neutrophils cling to the endothelial lining of capillaries in the injured area Marginated neutrophils on the vessel walls are able to detach and reenter the circulation when required.

pitting

process by which the spleen removes nuclei from red blood cells without destroying the erythrocytes occurs in splenic red pulp

Paroxysmal nocturnal hemoglobinuria (PNH)

rare and potentially life-threatening acquired clonal blood disorder with protean manifestations in which RBC membranes are highly vulnerable to damage by activated complement - especially at times of fever, acidosis, or hypoxia resulting chronic intravascular hemolysis is the hallmark of the classical hemolytic form of the disease release of free Hb contributes to most of its clinical manifestations

Chédiak-Higashi Syndrome

rare autosomal recessive disorder of the LYST gene, which encodes a protein responsible for lysosomal trafficking. Defective targeting of granules to the membrane results in large cytoplasmic granules that are unable to target to the plasma membrane in neutrophils, monocytes, and lymphocytes.

Hereditary Pyropoikilocytosis

rare, autosomal recessive disorder produces a molecular defect in spectrin and a partial spectrin deficiency characterized by thermal instability tend to experience severe hemolysis and anemia in infancy that gradually improves, evolving toward typical Hereditary Elliptocytosis later in life

clinical manifestations of Chédiak-Higashi Syndrome

recurrent bacterial infections of the skin, mouth, and respiratory tract; partial albinism; peripheral neuropathy; and mild bleeding disorders as a result of a deficiency in serotonin- and adenosine phosphate-containing granules in platelets. Defects in myelopoiesis result in neutropenia. Death usually occurs by 7 years of age because of infection. Advanced disease is characterized by lymphocytic tissue infiltrates and pancytopenia.

Erythrocyte membrane disorders

result from alterations in the quantity or quality (or both) of individual proteins and their dynamic interactions with each other

primary/hereditary hypercoagulable states

result from specific mutations or polymorphisms that lead to decreased levels of physiologic antithrombotic proteins or increased levels of procoagulant proteins.

α-Thalassemias

two major classes: α0, in which both α genes are inactivated (− −/); and α+, in which only one of the pair is defective because of α gene deletion or mutation (−α or ααT). clinical spectrum correlates well with the number of the affected α genes, that is, from normal to the loss of all four genes. 4 copies of the α-globin gene (two on each chromosome 16) -If one of the four genes is defective, the individual is termed a silent carrier -if 2 α-globin genes are defective, the individual is designated as having α-thalassemia trait. -if 3 α-globin genes are defective, the individual has Hb H (β4) disease—a mildly to moderately severe hemolytic anemia. -if 4 α-globin genes are defective, Hb Bart (γ4) disease with hydrops fetalis and fetal death results, because α-globin chains are required for the synthesis of HbF (fetal hemoglobin)

Shwachman-Diamond syndrome

usually begins as an isolated neutropenia but progresses to marrow failure and is also associated with pancreatic dysfunction and skeletal abnormalities. The responsible gene, the Shwachman-Bodian-Diamond syndrome gene (SBDS), is involved in the regulation of ribosomal RNA. These patients carry an increased risk for leukemic transformation.


Set pelajaran terkait

Anarchists, Blacklisting, Deflation, Fixed Costs, Holding Company, Horizontal Integration, Monopoly, Operating Costs, Tariff, Vertical Integration

View Set

LearningCurve 7a) A Healthy Time; Cognition

View Set

SOS HIGH SCHOOL HEALTH QUIZ 3: PERSONAL HYGIENE

View Set

Bullying, Interpersonal violence

View Set

Chapter 7.2 Body Planes, Directions, and Cavities

View Set

Simulation Lab 11.1: Module 11 Harden PC with Group Policy Editor

View Set

Evo Bio Ch. 12: Evolution of behavior

View Set

HS 8: Absolutism in Eastern Europe 1600-1740

View Set